Saturday 25 April 2015

CRNE – CGFNS



CRNE – CGFNS

Case Study : Grace, age 18 years, is scheduled for a cholecystectomy. Questions 1 and 2 relate to this scenario.
A.       After the nurse completes the preoperative teaching, Grace states, “If I lie still and avoid turning, I will avoid pain. Do you think this is a good idea?” What is the nurse’s best response?
A.             It is always a good idea to rest quietly after surgery.
B.              You need to turn from side to side every 2 hours.
C.              The doctor will probably order you to lie flat for 24 hours
D.             Why don’t you decide about activity after your return from the recovery room?

Answer: B
Rationale: To prevent venous stasis and improve muscle tone, circulation, and respiratory function, the client should be encouraged to move around after surgery. Pain medication will be administered to permit movement.

B.     Grace has returned from the operating room. When assessing her respiratory status as she recovers from general anaesthesia, it is of primary importance for the nurse to evaluate the patient’s ability to:
A.    Inhale voluntarily.
B.     Breathe deeply.
C.     Swallow.
D.    Speak.

Answer: C
Rationale: Anaesthesia interferes with the gag reflex until the gag reflex returns, the patient cannot swallow without a risk of aspiration.
Case study:As a nurse, you are taking care of Mr. Denny, age 59 years, who has been admitted with a myocardial infarction.Questions 3 to 6 related to this scenario.
C.    You observe that Mr. Denny is very sad and dejected. He states, “Life will never be the same.” How would you respond to him?
A.    This makes you really sad?
B.     Why do you think life will never be the same?
C.     Could you b a little more hopeful of your recover from this heart attack?
D.    You’re concerned when you think about how this will change your life?
Answer: D
Rationale: The response should be attune to the feelings of sadness and dejection as well as the content of the patient’s statement.
D.       What questions would you ask to determine his coping abilities?
A.     What could you have done to prevent this illness?
B.     How is this illness impacting you and your family?
C.     How can we take away your worries while you are in hospital?
D.    What are the worst challenges that you have faced?

Answer:   B
E.     Mr. Denny has been taking a diuretic, furosemide (Lasix). His serum potassium level is 3.2 mmol/L. Which assessment findings would confirm hypokalemia?
A.    Muscle weakness and a weak, irregular pulse.
B.     Diarrhea and cramps.
C.    Tetany and tremors.
D.    Headaches and poor tissue turgor.
Answer:A
Rationale: Muscle weakness and heart irregularities would be evident with hypokalemia.
F.     When Mr. Denny goes home, he enters a community heart rehabilitation program. What lifestyle changes will be reviewed?
A.           Reduced intake of unsaturated fats, anaerobic activity, increase fluids.
B.           Reduced intake of calcium, increased intake of sodium, rest periods.
C.           Reduced cholesterol levels, progressive activity levels, coping strategies.
D.          Increasing homocysteine levels, reducing weight, sedentary lifestyle
Answer: C
Rationale: Dietary changes, a progressive increase in activity, and effective coping strategies for stress reduction should all be included to assist in rehabilitation and reduce the risk of recurrence.

CGFNS
1.         The nurse is caring for a small child. Child abuse is suspected. The nurse understands that children are most frequently abused by a:
a)      Baby sitter
b)      Relative
c)      Teacher
d)     Casual acquaintance
ANSWER    B
in 90% of child physical abuse cases, the abuser is a relative whom he child trusts.
2.         A nurse and client working together for several weeks have developed a therapeutic relationship. As the nurse reminds the client that discharge is pending, the nurse can expect the client to most likely manifest symptoms of:
a.          Panic
b.         Grief
c.          Splitting
d.         Avoidance
ANSWER     B
the most common reaction to the experience of termination is grief. Panic may occur, but it is not most  likely.
3.         A female client who has been abusing her son is undergoing treatment to control her behavior. A statement by the client that indicates the development of some insight into her behavior as a parent would be:
a.       “I promise that I won’t get so angry when my son causes trouble again.”
b.      “Once my son gets straightened out, we should not have these problems.”
c.       “I think the root of the problem is when my husband comes home after drinking.”
d.      “If I feel angry at my son again, I’m going to go into the bedroom and punch a pillow.”
ANSWER     D
this plan for behavior shows potential for increased impulse control, which is important for the prevention of further abuse.
4.         The nurse is discussing electroconvulsive therapy (ECT) with a client who asks how long will it be before she feels better. The nurse explains that the beneficial effects of ECT usually occur within:
a.       One week
b.      Three weeks
c.       Four weeks
d.      Six weeks

ANSWER    A
beneficial effects of ECT usually are evident after the first several treatments. Since treatments are administered at intervals of 48 hours, these effects are apparent after one week of therapy.
5.         The nurse is caring for a client who is having a panic attack. Which symptom will the client least likely to exhibit?
a.          Bradycardia
b.         Choking
c.          Chest pain
d.         Fear of going crazy
ANSWER       A
 a panic attack stimulates the sympathetic nervous system, resulting in an increased heart rate. Other options are seen in panic attack.

Physiological Adaptation (1-10)

1. The major objective during the emergent phase of a burn is to:
a. Relieve pain
b. Prevent infection
c. Replace blood loss
d. Restore fluid volume
 D.    in the first 48 hours after a severe burn, fluid moves into the tissues surrounding the injured area. Fluid is also lost in drainage and from evaporation. This results in a decreased blood volume and could lead to shock.
2. A client with cirrhosis develops ascites. The nurse understands that the ascites is most likely the result of:
a. Impaired portal venous return
b. Inadequate secretions of bile salts
c. Excess production of serum albumin
d. Decreased interstitial osmotic pressure
A.             an enlarged liver impairs venous return, leading to increased portal vein hydrostatic pressure and a fluid shift into the abdominal cavity. Increased serum albumin causes hypervolemia not ascites.
3. A client with a history of heavy alcohol use develops portal hypertension and an elevated serum aldosterone level. The nurse should carefully observe the client for:
a. Chloride depletion and hypovolemia
b. Potassium retention and dysrhythmias
c. Sodium retention and fluid accumulation
d. Calcium depletion and pathologic fractures
 C.      aldosterone, a corticosteroid, causes sodium and water retention and potassium excretion by the kidneys.
4. A client has a tracheostomy tube attached to a tracheostomy collar for the delivery of humidified oxygen. The client will probably need suctioning primarily because the:
a. Humidified oxygen is saturated with fluid
b. Inner cannula of the tracheostomy tube irritates the mucosa
c. Tracheostomy tube interferes with the ability to cough effectively
d. Weaning process increases the amount of respiratory secretions
   C.     because the tracheostomy tube enters the trachea below the glottis, the client is unable to close the glottis to retain air in the lungs and raise the intrathoracic pressure and ten open the glottis to expel an explosive cough.
5. Two days following a myocardial infraction, a client’s temperature is elevated. This indicates:
a. Pneumonia
b. Tissue necrosis
c. Possible infection
d. Pulmonary infarction
B.    the body’s general inflammatory response to myocardial necrosis causes an elevation of temperature as well as leukocytosis within 24 to 48 hours.
6. When assessing a client for respiratory involvement during the first two days following a burn injury, the nurse should observe for sputum that is:
a.  Sooty
b.  Frothy
c.  Yellow
d.  Tenacious
A.   the mucous membrane of the respiratory tract may be charred after inhalation burns. This is evidence by the production of sooty sputum.

7. After one week a client with acute renal failure moves into the diuretic phase. During this phase the client must be carefully assessed for signs of:
a.  Hypovolemia
b.  Hyperkalemia
c.  Metabolic acidosis
d.  Chronic renal failure
 A.   in the diuretic phase, fluid retained during the oliguric phase is excreted and hypovolemia may occur. Diuresing is an indication of improved renal functioning, not progression into chronic renal failure.
8. The nurse evaluates that further discharge teaching is necessary when the postpartum client states, “I will call my health care provider if I experience:
a.  lochia which smells differently than normal
b.  A return to bright red vaginal discharge
c.  A temperature above 100.4 F, chills or perineal pain
d.  Any yellow-white discharge after 2 weeks
 D.   Lochia alba is creamy white or yellow in color and may last for 2-3 weeks following delivery. Other options must be reported.
9. A client is admitted to the nursing unit with a diagnosis of large bowel obstruction. On assessment, the nurse should expect to find:
a.   Distended abdomen, hypoactive bowel sounds
b.   Distended abdomen, normal bowel sounds in all quadrants
c.   Abdomen soft, bowel sounds present
d.   Abdomen soft, high-pitched bowel sounds
 A.   a client with a large bowel obstruction has a bowel full of air and fecal material unable to pass the blockage. The abdomen will be distended and bowel sounds may be hypoactive, absent or high pitched, but they will not be normal and present in all quadrants.
10. When children are dehydrated the nurse knows that they experience the greatest water loss through:
a.   Urination
b.   Respiration
c.   Perspiration
d.   Insensible water loss

D.  insensible water loss includes water lost through respiration (alveoli) and evaporation (integument). This is the most global response since it includes the second and third responses.

11. A nurse is caring for an elderly gentleman who suddenly goes into acute pulmonary edema. The most appropriate intervention is to administer:
a.   Oxygen and digoxin
b.   Furosemide and oxygen
c.   Oxygen and in a supine position
d.   Steroids and nitroglycerin
 B.  Furosemide is needed to remove excess fluid from the client’s circulation. Oxygen is a part of the immediate treatment.
12. The nurse is caring for a client with respiratory disorders. In assessing oxygenation, it is noted that when the PaO2 drops below 60 mmHg, the client will probably show signs of:
a.   Wheezing and hypotension
b.   Equal expiration and inspiration
c.   Diminished breath sounds and cyanosis
d.   Restlessness and tachycardia
 D.   tachycardia occurs as the heart compensates by increasing oxygen flow through the body, and restlessness results from decreased oxygen flow to the brain. Cyanosis is a late sign of hypoxemia.
13. When a client is in need of plasma proteins and clotting factors, which is the most appropriate transfusion?
a.   Packed red blood cells
b.   Fresh frozen plasma
c.   Cryoprecipitate
d.   Platelets
 B.  fresh frozen plasma contains plasma without red cells, plasma proteins, and clotting factors.
14. Following a thyroidectomy the nurse should carefully observe the clients for signs of thyroid storm. These include:
a.   Hypothermia
b.   Elevated serum calcium
c.   Sudden drop in pulse rate
d.   Rapid heart beat and tremors
 D.  thyroid storm refers to a sudden and excessive release of thyroid hormones, which cause pyrexia, tachycardia, and exaggerated symptoms of thyrotoxicosis. Surgery, infection, and ablation therapy can precipitate this life threatening condition. The pulse rate would be rapid in thyroid storm because of the sudden excessive release of thyroid hormones, which elevates the basal metabolic rate.
15. In a three-chamber underwater drainage system, the main purpose of the third chamber is to:
a.   Act as a drainage container
b.   Control the amount of suction
c.   Provide and air tight water seal
d.   Allow for escape of air bubbles

B.      the first chamber collects the drainage; the second chamber provides underwater seal; the third chamber controls the amount of suction.

 

Management of Care (6-15)
6. A registered nurse is preparing an assignment for a medical surgical unit. An unlicensed assistive person (UAP) should never be assigned to:
a. Assist physician with surgical procedure
b. Bathe a patient in traction
c. Ambulate a postoperative client
d. Bathe a cardiac client
B.           an unlicensed person is not a substitute for a registered nurse and should not assume assignments that require the skill of a registered nurse. Other options are within the scope of an unlicensed person.
7. An 89-year-old client is scheduled for elective surgery. Who should sign the surgical consent?
a. State-appointed guardian
b. Client’s next of kin or significant other
c. Hospital ethics committee member
d. Client
D.       a client who is deemed competent may sign his own consents regardless of age. Next of kin or significant other would only sign if the client were incompetent.
8. A psychiatric nurse is employed at a community health center and teaches stress-reduction exercise. This is an example of what level of care:
a. Primary prevention
b. Secondary prevention
c. Relapse prevention
d. Tertiary prevention
A.       primary prevention aims to alter risk factors that prevent or hinder the development of an illness. Secondary prevention attempts to reduce actual illness through early detection and treatment. Tertiary focuses on reducing the residual effects of an illness.
9. Which of the following nursing activities has the highest priority when a nurse is involved in the initial care of the newborn immediately after birth?
a. Maintaining warmth
b. Maintaining a patent airway
c. Attaching an identification band
d. Doing an Apgar score
B.        the first priority of the nurse in the initial care of the newborn is to maintain respirations. The infant should be placed in a modified Trendelenburg position to facilitate the drainage of mucous from the nasopharynx and trachea. The nurse may need to suction the nose and oropharynx with a bulb syringe to clear secretions
10. When documenting a client’s response to pain medication, which of the following statements would the nurse record?

a. Client appears comfortable and is resting without complaints at this time
b. Client appears to tolerate pain poorly, even after medication was given
c. Pain medication given with relief of pain 30 minutes after injection
d. Client reports pain to be a 3 on a 1 to 10 scale, with 10 being most severe pain
            D.  the data are reported and measured based on assessment with measurable outcomes.
11. The nursing diagnosis that is most appropriate for children with lead poisoning is:
              a. Risk for injury
b. Chronic pain
c. Altered nutrition
d. Unilateral neglect
A.   this is related to lead toxicity or buildup that causes fluid shifts into brain tissue, producing cell ischemia and destruction; this ultimately results in convulsions, mental retardation and death.
12. When the nurse is charting by exception in the client’s medical record, which of the following statement should the nurse record?
a. Client ambulated to bathroom without difficulty as before
b. Client reports pressure in chest when ambulating to bathroom
c. Five of client’s family members visiting at bedside this PM
d. Social worker in to discuss nursing home placement with family
 B.  chest pressure always needs to be noted, as well as activity. Option 1 is not necessary to document, if already charted previously. Visitors at the bedside may or may not be important to record. The social worker records his or her own visit and interaction with the client/family.
13. Before a 4-year-old child with a new colostomy is discharged, the nurse prepares a teaching plan for the parents that include telling them that:
a. An enterostomal therapist is available to assist with home care
b. They should try correcting the child’s poor eating habits at mealtime
c. Fluids should be limited between meals, although permitted at meals
d. The child should not take part in physical education when attending school
 A.  colostomy care may seem overwhelming to the parents, and it may reassure them to know that a therapist is available. Increased fluids are often needed to compensate for fecal fluid loss. Physical activity will probably not be limited.
14. When the nurse prepares to draw up 2 units of short-acting insulin and 3 units of long-acting insulin in the same syringe, the nurse should:
a. Inject air in the vial with the long-acting insulin first
b. Draw up the clear insulin first
c. Draw up either insulin first
d. Use a high-does insulin syringe
 A.    the air is injected into the long-acting insulin first. Air is then injected into the short-acting insulin is withdrawn. Then the long-acting insulin is withdrawn. It doe matter which insulin is drawn up first, because the nurse does not want to contaminate the short acting insulin with the long-acting insulin. A low-dose insulin syringe is used because a total of 5 units of insulin is needed.
15. The parents of an infant with cerebral palsy should be taught to:
a. Focus on cognitive rather than motor skills
b. Preserve muscle tone to prevent contractures
c. Maintain prolonged immobility of limbs with splints
d. Encourage strenuous exercise to build the infant’s muscle
B.      children with cerebral palsy are especially prone to muscle tone disorders, including spasticity, which can lead to contractures. In a therapeutic regimen there must be a balance between exercise and rest.

Pharmacology and Parenteral Therapy (26-35)
26. A client who uses birth control pills for contraception is to take a course of penicillin antibiotics to treat bronchitis. What will the nurse advise the client to do?
a. Increase the birth control pill dosage
b. Stop taking the birth control pills while taking the antibiotic
c. Stop taking the antibiotic as soon as symptoms disappear
d. Use an additional birth control method while taking antibiotics
D.  an additional birth control method should be used during penicillin therapy. Some antibiotics have been found to affect gastrointestinal flora and thereby interfere with absorption and metabolism of birth control pills.
27. A client arrives in the emergency department complaining of “crushing” chest pain. You anticipate a prescription for which of the following?
a. Nifedipine (Procardia)
b. Nizatidine (Axid)
c. Nortriptyline hydrochloride
d. Nystatin (Mycostatin)
A.          the nurse will anticipate a prescription for nifedipine (Procardia). Nifedipine is a calcium channel blocker that has coronary vasodilatory action. Vasodilation allows more oxygen to travel to the myocardial cells, thus relieving pain associated with ischemia.
28. A client is experiencing a hypertensive crisis. Blood pressure is 227/143 mmHg. Prescription reads: Diazoxide 100 mg IV. This medication will:
a. Decrease the heart rate by vagal stimulation
b. Directly relax arteriolar smooth muscle
c. Promote diuresis
d. Block calcium channels
C.          diazoxide (Hyperstat) is an antihypertensive (vasodilator) whose action directly relaxes arteriolar smooth muscles to lower blood pressure.
29. A client is receiving warfarin (Coumadin) following an episode of thrombophlebitis and is to be discharged on this medication. The nurse will instruct the client to:
a. Wear pants with an elastic waistband
b. Avoid crossing the legs
c. Consume foods high in vitamin K
d. Use birth control pills to prevent pregnancy
B.           clients with thrombophlebitis should avoid crossing their legs because this places weight on the popliteal space and decreases venous return to the heart.
30. Furosemide (Lasix) 40 mg is to be administered IV to a client with congestive heart failure and excessive weight gain. While the client is receiving Lasix, the nurse will closely observe for:
a. Elevated blood pressure
b. Presence of S3 on auscultation of the heart
c. Decrease in serum potassium level
d. Nausea and vomiting
C.       the nurse will observe for a decrease in serum potassium levels when clients are taking furosemide. Normal potassium levels are 3.5 to 5.5 mEq/L. the most significant side effects of furosemide include depletion of potassium, sodium, and hypotension. Clients should be specifically monitored for hypokalemia. Furosemide is a lop diuretic that increases excretion of water and electrolyte.
31. A client states, “My stool have been really dark lately.” Which of the following medications has probably caused this change?
a. Ferrous sulfate
b. Calcium carbonate
c. Ranitidine
d. Folic acid
 A.  ferrous sulfate has caused the change. The iron salts present in oral ferrous sulfate are excreted into the  stool, giving them a dark color. To prevent anxiety, clients should be informed that their stools are likely to be dark in color.
32. A client is brought to the emergency department after consuming an evening meal. The client’s tongue is swelling and obstructing the airway. The most appropriate therapy would be:
a. Oxygen via nasal cannula at 2L/minutes
b. Oxygen via humidified face masks at 100%
c. Oxygen via bag-valve mask at 100%
d. Oxygen via emergency tracheostomy at 100 %
 D.    oxygen via emergency tracheostomy at 100% is appropriate when treating airway obstruction. Some anaphylactic allergies may cause the tongue to swell. If airway obstruction occurs, the airway must be opened below the site of obstruction and oxygen delivered.
33. A client is receiving ritodrine hydrochloride (Yutopar) IV. With the initiation of this drug, it is most important to assess for any adverse reactions that could affect:
a. Uterine function
b. Gastrointestinal function
c. Central nervous system function
d. Cardiac function
 D.    the nurse should assess for adverse reactions affecting cardiac function. The administration of ritodrine hydrochloride is associated with cardiovascular side effects such as tachycardia, palpitations, premature ventricular contractions, and widening pulse pressure. These effects are the most important to assess since they may be life threatening.
34. A client newly diagnosed with diabetes mellitus is instructed by the physician to obtain glucagon for emergency home use. The client asks a home care nurse about the purpose of the medication. The nurse instruct the client that the purpose of the medication is to treat:
a. Hypoglycemia from insulin overdose
b. Hyperglycemia from insufficient insulin
c. Lipoatrophy from insulin injections
d. Lipohypertrophy from inadequate insulin absorption
 A.   Glucagon is used to treat hypoglycemia resulting from insulin overdose. The family of the client is instructed in how to administer the medication in an unconscious client. Arousal usually occurs within 20 minutes of glucagons injection. Once consciousness has been regained, oral carbohydrates should be given. Lipoatrophy and lipohypertophy result from insulin injection
35. The nurse administers oral thyroid hormone to an infant with hypothyroidism. For which of the following signs of overdose should the nurse observe the infant?
a. Tachycardia, fever, irritability, and sweating
b. Bradycardia, cool skin temperature, and dry scaly skin
c. Bradycardia, fever, hypotension, and irritability
d. Tachycardia, cool skin temperature, and irritability
A.    the infant experiencing an overdose of thyroid replacement hormone exhibits tachycardia, fever, irritability, and seating. Bradycardia, cool skin temperature, and dry scaly skin are signs of hypothyroidism.

Reduction of Risk Potential (31-40)
31. Three days following of a spica cast, a child has a temperature of 101.4 F. suspecting an infection; the nurse should first assess the child for:
a. Rapid irregular respirations
b. A foul smelling coming from the cast
c. Any complaints of tingling in the toes
d. The presence of itching around the top of the cast
C.       this may be indicative of an infection under the cast and would probably cause a fever. Tingling of the toes may indicate cardiovascular impairment
32. The nurse is aware that the aim of palliative surgery for children with Tetralogy of Fallot is to directly increase the blood flow to the:
a. Brain
b. Lungs
c. Myocardium
d. Right ventricle
 B.  this defect causes blood to bypass the lungs; surgery increases blood flow to the lungs.
33. After surgery to create an ileal conduit, a client is admitted to the postanesthesia care unit. During the first hour of the postoperative period, the nurse should notify the physician if:
a. Vomiting occurs
b. The stoma is swollen
c. No urine output
d. Bowel sounds are diminished
D.    urine should drain usually from the conduit because there is no sphincter control unless a continent conduit is created. Vomiting is a common occurrence after anesthesia.
34. When a client returns from surgery after thyroidectomy, the nurse should position the client:
a. Prone with the head turned to one side
b. Supine with the knees flexed at 10 degrees
c. Lateral with the head slightly flexed and elevated
d. Supine with the head continuously hyperextended
 C. elevation decreases edema, promotes drainage, and facilitates respirations, while slight flexion prevents strain on the suture line.
35. A client with cardiac problems is to have Holter monitor at home. The nurse should instruct the client to:
a. Keep a dairy of activities
b. Stay away from microwave ovens
c. Avoid taking any nitroglycerin that day
d. Take both blood pressure and pulse every 2 hours
A.    the purpose of monitoring is to correlate dysrhythmias with the client’s reported activity. The client should take nitroglycerin as needed and not it in the activities of dairy. A microwave oven would have no effect on the Holter monitor and would not affect the readings.
36. A client has surgery to remove a stone from the common bile duct. The nurse is aware that the bile flow into the duodenum has been re-established after biliary surgery when:
a. The liver is no longer tender
b. Stools are normal brown in color
c. Colic is absent after ingestion of fats
d. The serum bilirubin level returns to normal
B.   the return of brown color to the stool indicates that bile is entering the duodenum and being converted to urobilinogen by bacteria.
37. To help limit a common complication following the repair of an inguinal hernia, the nurse should:
a. Apply an abdominal binder
b. Place a support under the scrotum
c. Encourage a high-carbohydrate diet
d. Have the client cough and deep breathe frequently
B.   after inguinal hernia repair, the scrotum commonly becomes edematous and painful. Drainage is facilitated by elevating the scrotum on rolled linen or using a scrotal support.
38. A woman is admitted to the hospital for an elective cesarean birth. To prepare the client for surgery, the nurse:
a. Shaves the perineal area
b. Administers a tap water enema
c. Inserts an indwelling catheter
d. Assists the client with an antiseptic shower
C.    an indwelling catheter will keep the bladder empty, protecting it from injury during surgery. An abdominal shave would be more appropriate, a cleansing enema is usually not required because surgical procedures do not involve the intestines.
39. When caring for a child with vitamin K deficiency, the nurse should be alert for which of the following sequelae?
a. Prolonged bleeding time
b. Visual disturbance
c. Anorexia
d. Fatigue
A.     a deficiency in vitamin K can lead to increased bleeding times since vitamin K is necessary for the formation of several clotting factors.
40. When counseling parents about lead poisoning prevention, the nurse should include which of the following in her teaching plan?
a. Dry cleaning techniques are helpful
b. Toys and pacifiers should be cleaned weekly
c. Water should be run for 5 minutes prior to use
d. Cold water should be used for consumption
D.    lead concentrations are lower in cold water than in hot water. Tap water should be run from 30 seconds to 2 minutes prior to use. Wet cleaning methods more effectively remove lead containing dust.

Basic Care and Comfort (1-15)
1. A client with a history of chronic arterial occlusive disease complains of pain in the legs while walking. Which would be the best action for the nurse to implement?
a. Continue the exercise
b. Give the client a break and then resume the exercise
c. Stop the exercise and administer a narcotic
d. Place the client on bedrest and notify the physician
B.    progressive exercise needs to be initiated to develop collateral circulation. Pain is due to ischemic muscle and is relieved by resting. 
2. Which of the following laboratory results best indicates to the nurse the nutritional status of a 45-year-old man with left-sided weakness from a CVA?
a. Blood glucose 80 mg/dl
b. Albumin 3.0 g/dl
c. Serum cholesterol 200 mg/dl
d. Hematocrit 46%
 B.     normal level is 3.5 – 5.o g/dl; reduced levels indicate catabolism; 3.5 g/dl indicates malnutrition.
3. The nurse supervises the nurse’s aide providing care to a patient with a chest tube on the left side, which is attached to a three-chamber water-seal drainage system (Pleur-evac). The patient should be placed:
a. on his left side
b. Flat with his feet elevated
c. On his back
d. With his head elevated
D.       this is a low Fowler’s position, patient’s trunk raised to form an angle of 15-30 degree, this will maximize chest expansion and increase a respiratory function.
4. The school nurse is performing health screening for scoliosis on a group of sixth graders. The nurse would be most concerned if:
 a. A child complains of a painful right knee
b. A child’s feet turn inward
c. A child shifts his weight from the right foot to the left foot
d. A child’s left shoulder is higher than the right shoulder
 D.      this is a symptom of scoliosis. The nurse should also observe for asymmetry or prominence of rib cage and scapulae, curvature of spine.
5. A 37-year-old man is undergoing testing for amyotrophic lateral sclerosis (ALS). The nurse would expect the client to exhibit:
a. Incontinence of bowel and bladder
b. Difficulty swallowing
c. Paresthesia of the face
d. Disorientation of time and place
B.   a client with ALS has a difficulty with dysphagia and aspiration. Other options are not seen with ALS
6. A 60-year-old woman comes to the hospital with a fracture of her right femur. The nurse would be most concerned if:
a. The patient’s BP changes from 120/72 to 130/80
b. The patient complains of abdominal pain and eructates frequently
c. The patient is incontinent of urine and stool
d. The patient plucks at the bed covers and talks to the wall
 D.   confusion, restlessness, agitation are symptoms of fat embolism, seen with fractures of long bones; other symptoms are dyspnea, fever petechial skin rash.
7. The nurse is caring for a patient with a casted left leg. Which of the following exercises should the nurse recommend?
a. Passive exercise of the affected limb
b. Quadriceps setting of the affected limb
c. Active ROM exercises of the unaffected limb
d. Passive exercise of the upper extremities
 B.   isometric exercise; contraction of muscle without movement of joint; maintains strength while in cast.
8. The nurse is aware that a 10-month-old infant on a regular diet could be fed:
a. Applesauce, carrots, chicken, and formula
b. Pears, green beans, turkey, and whole milk
c. Bananas, sweet potatoes, ham, and formula
d. Peaches, corn, cottage cheese, and whole milk
 A.    these easily digested foods have usually been introduced by 10 months of age; breast milk or formula is recommended for the first year of life.
9. In management of a child with newly diagnosed with chronic celiac disease, the primary nursing goal is to:
a. Prevent celiac crisis and resulting complications
b. Prevent complications from respiratory involvement
c. Teach the parents to control the diet to promote normal growth
d. Help the parents and child adjust to the lifelong dietary restrictions
 D.   lifelong adherence to dietary restrictions prevents complications and celiac crisis. Celiac crisis usually develops as a result of non-adherence to the diet, so adherence would be a priority goal.
10. The regulation of diabetes in a newly diagnosed juvenile is best accomplished by:
a. Insulin, dietary control, and exercise
b. Dietary control, exercise, and urine testing
c. Dietary control, exercise, and blood glucose monitoring
d. Oral hypoglycemic agents, dietary control, and exercise
A.      most juveniles are type I diabetics and have little or no endogenous insulin; diet control and exercise reduce the amount of exogenous insulin needed.
11. When examining the throat of a 5-year-old, the nurse should position a tongue blade to the side of the child’s tongue primarily to avoid:
a. Eliciting the gag reflex
b. Obstructing the airway
c. Hurting any of the teeth
d. Interfering with the visual examination
A.    the gag reflex is elicited by pressing on the posterior pharynx, resulting in glossopharyngeal stimulation; inserting the tongue blade on the side of the mouth limits this stimulation
12. The advantage of a gastrostomy tube feeding over a nasogastric tube is that:
a. There is less chance of aspiration
b. The procedure does not require gravity
c. The client can self-administer the feeding
d. More tube feeding mixture can be given each time
A.    a gastrostomy is an opening made directly through the abdominal wall into the stomach. When tube feedings are given via this route, they bypass the upper GI tract and reduce the risk of tracheal aspiration. Clients can be taught to feed themselves with either method.
13. When preparing a teaching plan for a client on hemodialysis, the nurse recalls that a substance that passes through the semipermeable membrane during hemodialysis is:
a. RBCs
b. Sodium
c. Glucose
d. Bacteria
B.   sodium is an electrolyte that can pass through the semipermeable membrane during hemodialysis. RBCs, glucose and bacteria do not pass through the semipermeable membrane during hemodialysis.
14. The major nursing problem in caring for a client with hyperthyroidism would be:
a. Providing sufficient rest
b. Modifying hospital routines
c. Providing an adequate diet
d. Keeping the bed linen neat
A.   promotion of rest to reduce metabolic demands is a challenging but essential task with a client who has hyperthyroidism. The diet can be increased to meet the metabolic demands; the client usually has an excellent appetite.
15. When a client with kidney shutdown complains of thirst, the nurse should offer:
a. Carbonated soda
b. Sour ball candy
c. A glass of milk
d. A bowl of clear soup
B.    sucking on candy will relieve thirst and provide calories without supplying extra fluid. Fluids must be restricted.

Growth and Development (6-10)

6. The nurse should advise the pregnant client to use which body position to enhance cardiac output and renal function?
a. Right lateral
b. Left lateral
c. Supine
d. Semi-Fowler’s
B.  the left lateral position shifts the enlarged uterus away from the vena cava and aorta, enhancing cardiac output, kidney perfusion and kidney function. The right lateral position and semi-Fowler’s positions don’t alleviate pressure of the enlarged uterus on the vena cava. The supine position reduces sodium and water excretion because the large uterus compresses the vena cava and aorta; this decreases cardiac output, leading to decreased renal blood flow, which in turn impairs kidney function.
7. A client who used heroin during her pregnancy delivers a neonate. When assessing the neonate, the nurse expects to find:
a. 1 Lethargy 2 days after birth
b. 2. Irritability and poor sucking
c. 3. A flattened nose, small eyes and thin lips
d. 4. Congenital anomalies
C.     neonates of heroin-addicted mothers are physically dependent on the drug and experience withdrawal when the drug is no longer supplied. Signs of heroin withdrawal include irritability, poor sucking, and restlessness. Lethargy isn’t associated with neonatal heroin addiction. A flattened nose, small eyes, and thin lips are seen in infants with fetal alcohol syndrome. Heroin use during pregnancy hasn’t been linked to specific congenital anomalies.
8. The nurse places a neonate with hyperbilirubinemia under a phototherapy under a phototherapy lamp. The goal of phototherapy is:
a. To prevent hypothermia
b. To promote respiratory stability
c. To decrease the serum conjugated bilirubin level
D.    the goal of phototherapy is to reduce serum unconjugated bilirubin level because a high level may lead to bilirubin encephalopathy (kernicterus). Phototherapy doesn’t prevent hypothermia or promote respiratory stability. It has no effect on conjugated bilirubin, a water-soluble substance excreted easily in urine and stools.d. To decrease the serum unconjugated bilirubin level

9. The nurse is assessing the sexual development of a preteenage girl. What is the first sign of sexual maturation in females?
a. Onset of menstruation
b. Breast development
c. Appearance of pubic hair
d. Appearance of axillary hair
A.     the first sign of sexual maturation in females is the development of breast buds (elevation of the nipples and areolae). Then sexual development progresses, causing the appearance of pubic hair and axillary and the onset of menstruation.
10. The nurse performs a gestational age assessment on a newly delivered infant. Which of the following assessments is included in the gestational age assessment?
a. Overriding cranial bones
b. Lanugo
c. Tonic neck reflex
d. Gelatinous umbilical cord
B.     lanugo is the fine downy hair that covers the skin in preterm infants. The lanugo decreases as gestational age increases. The amount of lanugo is greatest at 28-30 weeks gestation and then disappears first from the face and then from the trunk and extremities.

Coping and Adaptation (1-5)
1. A young man with Hodgkin’s disease has been readmitted to the hospital because of his aggressive disease that is unresponsive to multiple therapies. Death appears imminent. One goal for this client is to:
a. Reduce feeling of isolation
b. Reduce fear of pain
c. Reduce fear of more aggressive therapies
d. Reduce feelings of social intimacy
A.               terminally clients most often describe feelings of isolation because they feel ignored. The terminally ill client may sense any discomfort that family and friends feel in the client’s presence. Nursing interventions include spending time with the client, encouraging discussion about feelings and answering question openly and honestly.
2. A family may request to have a client of Vietnamese descent transferred to die at home because it is traditionally believed that:
a. It is disloyal to their loved one to be left in the hospital
b. The hospital cannot be trusted
c. The family can provide more comfort at home
d. Reincarnation can provide will not occur in the hospital
 C.     the traditional belief of Vietnamese Americans is that the family can provide more comfort for their loved one at home. It is not seen as being disloyal if their loved one dies in the hospital. The request is not based on a feeling that the hospital cannot be trusted. The Vietnamese Americans accept death as a part of life and do not think hat reincarnation is prevented in the hospital.
3. An adolescent client hospitalized with a recent renal transplant expresses frustration and states, “I wish I could be out with my friends.” Which nursing diagnosis most accurate describes this behavior?
a. Hopelessness
b. Social isolation
c. Ineffective denial
d. Dysfunctional grieving
 B.   according to a psychosocial model of development, the adolescent is very much involved with a peer   group. The adolescent is expressing feelings of loneliness and frustration.
4. A client in participating in bereavement group after her abusive spouse recently died. The nurse, in discussing grieving, identifies that this process is prolonged when the survivor has:
a. Feelings of guilt
b. Difficulty with anger
c. A sense of relief regarding the death
d. Ambivalence about the relationship with the deceased
 A.    if there are guilty feelings about the deceased, the client may have difficulty dealing with the death. This is especially true if relationship was an abusive one.
5. During an individual session, the client discloses to the nurse that she suspects her husband is having an extramarital affair. In frustration, the client turns to the nurse and asks, “Do you think I should divorce him?” The most therapeutic response is:
a. “Well, it really doesn’t matter what I think.”
b. “That sounds like a good decision, given his behavior.”
c. “Sometimes divorce is necessary.”
d. “What do you want to do?”
D.       reflection is an effective communication style, which diverts the client’s question to encourage expression of  thought and feeling.


Safety and Infection Control (1-5)
1. During a home visit to an elderly client who has just had a total hip replacement and is ambulatory with the use of a walker, the nurse assesses the client’s safety. Which of the following observations would concern the nurse the most?
a. The client uses a high-rise seat on the toilet
b. All rooms have polished hard wood floors with area rugs
c. The client has no cellular phone
d. The client sleeps with the walker next to the bedside
 B.    polished floors and scatter rugs pose a safety hazard to this elderly client.
2. To prevent re-programming of an artificial cardiac pacemaker, the client should be instructed to avoid being near:
a. Chest X-ray machine
b. High-output electrical generators
c. Microwave ovens’
d. Metal detectors at airports
 B.   the clients needs to avoid sources of strong electromagnetic fields, such as magnets, telecommunications transmitters, and electrical generators. Clients with pacemaker can have chest x-rays when necessary. Microwave ovens will not interfere with the pacemaker. Metal detectors are safe.
3. Which special precaution must the nurse take when assisting a client with self-monitoring of blood glucose?
a. Give the client a machine for his use only
b. Wear gloves when performing the test
c. Rinse the lancet between uses
d. Recalibrate the glucometer before each use
 B.   the nurse should wear gloves and protecting clothing if necessary, any time contact with blood or body fluids is anticipated
4. When a client has had surgery for a ruptured appendix, the nurse should document the wound as;
a. Clean
b. Clean-contaminated
c. Contaminated
d. Infected
 D.   when the client has a ruptured appendix, the nurse should document the surgical wound as an infected wound, because bacterial organisms are present in the wound and there are signs of infections
5. After abdominal surgery, a priority nursing intervention for a young infant with an IV is:
a. Administering oral fluids
b. Limiting handling by the parents
c. Weighing the diapers after voiding
d. Placing elbow restraints on both arms
D.         safety is a priority; the infant may inadvertently dislodge the circulatory access. Parent-infant contact should be encouraged.

Endocrine NCLEX Practice Questions (1-30)
1. The nurse is performing an admission assessment on a client admitted to the hospital with a diagnosis of pheochromocytoma. The nurse prepares to implement what action to assess for the principal manifestation associated with this disorder?
a) checks the client’s pupils
b) checks the peripheral pulses
c) takes the client’s blood pressure
d) assesses for the presence of peripheral edema
 C.  Pheochromocytoma is a catecholamine-secreting tumor that is usually located in the adrenal medulla. Hypertension is the principal manifestation associated with pheochromocytoma, and it can be persistent, fluctuating, intermittent, or paroxysmal. The blood pressure status would be assessed by taking the client’s blood pressure. The assessments in options A, B, and D are not associated with this disorder.
2.  Regular insulin by continuous intravenous (IV) infusion is prescribed for a client with a blood glucose level of 700 mg/dL. The nurse plans to:
a) mix the solution in 5% dextrose
b) change the solution every 6 hours
c) infuse the medication via an electronic infusion pump
d) titrate the infusion according to the client’s urine glucose levels
 C.  Insulin is administered via an infusion pump to prevent inadvertent overdose and subsequent hypoglycemia. There is no reason to change the solution every 6 hours. Dextrose is added to the IV infusion once the serum glucose level reaches 250 mg/dL to prevent the occurrence of hypoglycemia. Administering dextrose to a client with a serum glucose level of 700 mg/dL would counteract the beneficial effects of insulin in reducing the glucose level. Glycosuria is not a reliable indicator of the actual serum glucose levels because many factors affect the renal threshold for glucose loss in the urine.
3.   The nurse is developing a plan of a care for a client with diabetic ketoacidosis (DKA). The nurse includes which intervention in the plan?
a) assess for fluid overload
b) limit family visitation time
c) ambulate the client every 2 hours
d) maintain side rails in the upright position
 D.   The client with DKA may experience a decrease in the level of consciousness (LOC) secondary to acidosis. Safety becomes a priority for any client with a decreased LOC, thus requiring the use of side rails to prevent fall injuries. The client will experience fluid loss (dehydration) rather than overload and may be too ill to ambulate. Family visitation is helpful for both the client and family to assist with psychosocial adaptation.
4.    A client undergoes a subtotal thyroidectomy. The nurse ensures that which priority item is at the client’s bedside upon arrival from the operating room?
a) an apnea monitor
b) a suction unit and oxygen
c) a blood transfusion warmer
d) an ampule of phytonadione (vitamin K)
 B.  Following thyroidectomy, respiratory distress can occur from tetany, tissue swelling, or hemorrhage. It is important to have oxygen and suction equipment readily available and in working order if such an emergency were to arise. Apnea is not a problem associated with thyroidectomy, unless the client experienced a respiratory arrest. Blood transfusions can be administered without a warmer, if necessary. Vitamin K would not be administered for a client who is hemorrhaging, unless deficiencies in clotting factors warrant its administration.
5.   The nurse is preparing the bedside for a postoperative parathyroidectomy client who is expected to return to the nursing unit from the recovery room in 1 hour. The nurse ensures that which piece of medical equipment is at the client’s bedside?
a) cardiac monitor
b) tracheotomy set
c) intermittent gastric suction
d) underwater seal chest drainage system
B.      Respiratory distress caused by hemorrhage and swelling and compression of the trachea is a primary concern for the nurse managing the care of a postoperative parathyroidectomy client. An emergency tracheotomy set is always routinely placed at the bedside of the client with this type of surgery, in anticipation of this potential complication. Options A, C, and D are not specifically needed with the surgical procedure.
6. A client is diagnosed with hypothyroidism and is scheduled to begin taking thyroid supplements. The nurse instructs the client about the medication. Which statement by the client would indicate the need for further instructions?
a) I need to take my daily dose every night at bedtime
b) I need to notify my physician if I develop any chest pain
c) I may experience some gastrointestinal problems, such as diarrhea
d) I need to speak to my physician when I begin to plan for parenthood
         A.  The client is instructed to take the medication in the morning to prevent insomnia. If the client experiences any chest pain, it may indicate an overdose, and the physician needs to be notified. Gastrointestinal complaints from thyroid supplements include increased appetite, nausea, and diarrhea. The dose needs to be adjusted if the client is pregnant or plans to get pregnant.
7.  The clinic nurse instructs a client with diabetes mellitus about how to prevent diabetic ketoacidosis on days when the client is feeling ill. Which statement by the client indicates the need for further instructions?
a) I need to stop my insulin if I am vomiting
b) I need to eat 10 to 15 g of carbohydrates every 1 to 2 hours
c) I need to call my physician if I am ill for more than 24 hours
d) I need to drink small quantities of fluid every 15 to 30 minutes
 A.  The client needs to be instructed to take insulin, even if he or she is vomiting and unable to eat. It is important to self-monitor blood glucose more frequently during illness (every 2 to 4 hours). If the premeal blood glucose is more than 250 mg/dL, the client should test for urine ketones and contact the physician. Options B, C, and D are accurate interventions.
8. The nurse is instructing a client with diabetes mellitus regarding hypoglycemia. Which statement by the client indicates the need for further instructions?
a) hypoglycemia can occur at any time of the day or night
b) I can drink 6 to 8 ounces of milk if hypoglycemia occurs
c) if I feel sweaty or shaky, I might be experiencing hypoglycemia
d) if hypoglycemia occurs, I need to take my regular insulin as prescribed
 D.   Insulin is not taken as a treatment for hypoglycemia, because the insulin will lower the blood glucose level. Hypoglycemic reactions can occur at any time of the day or night. If a hypoglycemic reaction occurs, the client will need to consume 10 to 15 g of carbohydrate; 6 to 8 ounces of milk contains this amount of carbohydrate. Tremors and diaphoresis are signs of mild hypoglycemia.
9. The nurse develops a plan of care for an older client with diabetes mellitus. The nurse plans to first:
a) structure menus for adherence to diet
b) teach with videotapes showing insulin administration to ensure competence
c) encourage dependence on others to prepare the client for the chronicity of the disease
d) assess the client’s ability to read label markings on syringes and glucose monitoring equipment
 D.    The nurse first assesses the client’s ability for self-care. Structuring menus for the client promotes dependence. Allowing the client to have hands-on experience rather than teaching with videos is more effective. Independence should be encouraged.
10. A nurse is teaching a client who had been newly diagnosed with diabetes mellitus about blood glucose monitoring. The nurse teaches the client to report glucose levels that consistently exceed:
a) 150 mg/dL
b) 200 mg/dL
c) 250 mg/dL
d) 350 mg/dL
C.        The client should be taught to report blood glucose levels that exceed 250 mg/dL, unless otherwise instructed by the physician. Options A and B are low levels that do not require physician notification. Option D is a high value.

11. The nurse is reviewing home-care instructions with an older client who has type 1 diabetes mellitus and a history of diabetic ketoacidosis (DKA). The client’s spouse is present when the instructions are given. Which statement by the spouse indicates that further teaching is necessary?
a) if he is vomiting, I shouldn’t give him any insulin
b) I should bring him to physician’s office if he develops a fever
c) if the grandchildren are sick, they probably shouldn’t come to visit
d) I should call the doctor if he has nausea or abdominal pain lasting for more than 1 or 2 days
 A.    Infection and the stopping of insulin are precipitating factors for DKA. Nausea and abdominal pain that last more than 1 or 2 days need to be reported to the physician, because these signs may be indicative of DKA.
12. The nurse provides home-care instructions to a client with Cushing’s syndrome. The nurse determines that the client understands the hospital discharge instructions if the client makes which statement?
a) I need to eat foods low in potassium
b) I need to check the color of my stools
c) I need to check the temperature of my legs twice a day
d) I need to take aspirin rather than Tylenol for a headache
 B   Cushing’s syndrome results in an increased secretion of cortisol. Cortisol stimulates the secretion of gastric acid, and this can result in the development of peptic ulcers and gastrointestinal bleeding. The client should be encouraged to eat potassium-rich foods to correct the hypokalemia that occurs with this disorder. Cushing’s syndrome does not affect temperature changes in the lower extremities. Aspirin can increase the risk for gastric bleeding and skin bruising.
13.  The nurse in an outpatient diabetes clinic is monitoring a client with type 1 diabetes mellitus. Today’s blood work reveals a glycosylated hemoglobin level of 10%. The nurse creates a teaching plan on the basis of the understanding that this result indicates which of the following?
a) a normal value that indicates that the client is managing blood glucose control well
b) a value that does not offer information regarding that client’s management of the disease
c) a low value that the client is not managing blood glucose control very well
d) a high value that indicates the client is not managing blood glucose control very well
 D.    Glycosylated hemoglobin is a measure of glucose control during the 6 to 8 weeks before the test. It is a reliable measure for determining the degree of glucose control in diabetic clients over a period of time, and it is not influenced by dietary management a day or two before the test is done. The glycosylated hemoglobin level should be 7.0% or less, with elevated levels indicating poor glucose control.
14. The nurse is instructing a client with type 1 diabetes mellitus about the management of hypoglycemic reactions. The nurse instructs the client that hypoglycemia most likely occurs during what time interval after insulin administration?
 A.   Insulin reactions are most likely to occur during the peak time after insulin administration, when the medication is at its maximum action. Peak action depends on the type of insulin, the amount administered, the injection site, and other factors.
15. The nurse is caring for a client with type 1 diabetes mellitus. Because the client is at risk for hypoglycemia, the nurse teaches the client to:
a) monitor the urine for acetone
b) report any feelings of drowsiness
c) keep glucose tablets and subcutaneous glucagon available
d) omit the evening dose of NPH insulin if the client has been exercising
C.    Glucose tablets are taken if a hypoglycemic reaction occurs. Glucagon is administered subcutaneously or intramuscularly if the client loses consciousness and is unable to take glucose by mouth. Glucagon releases glycogen stores and raises the blood glucose levels of hypoglycemic clients. Family members can be taught to administer this medication and possibly to prevent an emergency department visit. Acetone in the urine may indicate hyperglycemia. Although signs of hypoglycemia need to be taught to the client, drowsiness and coma are not the initial and key signs of this complication. The nurse would not instruct a client to omit insulin.
16. A client with diabetes mellitus has received instructions about foot care. Which statement by the client would indicate that the client needs further instructions?
a) I’ll trim my nails straight across after my bath
b) my feet should be inspected daily using a mirror
c) the cuticles of my nails must be cut to prevent overgrowth
d) cotton stockings should be worn to absorb excess moisture
 C.   Trimming or cutting the cuticles of the nails can lead to injury to the foot by scratching the skin. Even small injuries can be dangerous to the client with diabetes mellitus who has decreased peripheral vascular circulation. A manicure stick can be used to gently push the cuticle back under the nail. Nails can be cut straight across; and after a bath is the best time, because the nails are softest then. White cotton stockings are best, and the client needs to inspect the feet daily. The client can use a mirror for those areas that are difficult to inspect.
17.    The nurse has taught a client about the signs and symptoms and treatment of hyperglycemia. Which statement by the client reflects an accurate understanding?
a) I may become diaphoretic and faint
b) I may notice that I have dry skin and increased urination and thirst
c) I should restrict my fluid intake if my blood glucose level is more than 250 mg/dl
d) I need to take an extra diabetic pill if my blood glucose level is more than 300 mg/dL
      B.  Dry skin, polyuria (excess urination), and polydipsia (excess thirst) are classic symptoms of hyperglycemia. Dry skin occurs as a result of dehydration related to the polyuria. Polydipsia occurs as a result of fluid loss. Diaphoresis is associated with hypoglycemia. Clients should not take extra oral hypoglycemic agents to reduce an elevated blood glucose level. A client with hyperglycemia becomes dehydrated as a result of the osmotic effect of elevated glucose; therefore, the client must increase fluid intake.
18. A client with the diagnosis of hyperparathyroidism says to the nurse, “I can’t stay on this diet. It is too difficult for me.” When intervening in this situation, how should the nurse respond?
a) Why do you think you find this diet plan difficult to adhere to?
b) it really isn’t difficult to stick to this diet. Just avoid milk products
c) you are having difficult time staying on this plan. Let’s discuss this
d) it is very important that you stay on this diet to avoid forming renal calculi
 C.   By paraphrasing the client’s statement, the nurse can encourage the client to verbalize emotions. The nurse also sends feedback to the client that the message was understood. An open-ended statement or question such as this prompts a lengthy response from the client. Option A requests information that the client may not be able to express. Option B devalues the client’s feelings. Option D gives advice, which blocks communication.
19. The nurse is caring for a client with newly diagnosed type 1 diabetes mellitus. To develop an effective teaching plan, it would be most important for the nurse to assess the client for:
a) knowledge of the diabetic diet
b) expression of denial of having diabetes
c) fear of performing insulin administration
d) feelings of depression about lifestyle changes
 B.   When diabetes mellitus is first diagnosed, the client may go through the phases of grief: denial, fear, anger, bargaining, depression, and acceptance. Denial is the phase that is the most detrimental to the teaching and learning process. If the client is denying the fact that he or she has diabetes, then he or she probably will not listen to discussions about the disease or how to manage it. Denial must be identified before the nurse can develop a teaching plan.
20. A client with newly diagnosed type 1 diabetes mellitus has been seen for 3 consecutive days in the emergency department with hyperglycemia. During the assessment, the client says to the nurse, “I’m sorry to keep bothering you every day, but I just can’t give myself those awful shots.” The nurse makes which therapeutic response?
a) I couldn’t give myself a shot either
b) you must learn to give yourself the shots
c) let me see if the doctor can change your medication
d) I’m sorry that you are having trouble with your injections. Has someone given you instructions about how to perform them?
D.    It is important to determine and deal with a client’s underlying fear of self-injection. The nurse should determine whether a knowledge deficit exists. Positive reinforcement should occur rather than focusing on negative behaviors (option A). Demanding that the client perform a behavior or skill is inappropriate (option B). The nurse should not offer a change in regimen that cannot be accomplished (option C).
21. The nurse requests that a client with diabetes mellitus ask his or her significant other(s) to attend an educational conference about the self-administration of insulin. The client questions why significant others need to be included. The nurse’s best response would be:
a) family members can take you to the doctor
b) family members are at risk of developing diabetes
c) nurses need someone to call and check on a client’s progress
d) clients and families often work together to develop strategies for the management of diabetes
 D.    Families and significant others may be included in diabetes education to assist with adjustments of the diabetic regimen. Although options A and B may be accurate, they are not the most appropriate responses. Option C devalues the client, disregards the issue of independence, and promotes powerlessness.
22. A male client is admitted to the hospital with diabetic ketoacidosis (DKA). The client’s daughter says to the nurse, “My mother died last month, and now this. I’ve been trying to follow all of the instructions form the doctor, but what have I done wrong?” The nurse makes which response to the client’s daughter?
a)         tell me what you think you did wrong
b)          maybe we can keep your father in the hospital for a while longer to give you a rest
c)         you should talk to the social worker about getting you someone at home who is more capable with managing a diabetic’s care
d)        an emotional stress such as your mother’s death can trigger DKA in a diabetic client, even though the prescribed regimen is being followed
D.    initiate the physiological mechanism of DKA. Options A and C substantiate the daughters’ feelings of guilt and incompetence. Option B is not a cost-effective intervention.
23. A client with hyperaldosteronism has developed renal failure and says to the nurse, “This means that i will die very soon.” The nurse makes which appropriate response to the client?
a)    you will do just fine
b)   what are you thinking about?
c)   you sound discouraged today
d)   I read that death is a beautiful experience
 C.   Option C uses the therapeutic communication technique of reflection, and it both clarifies and encourages the further expression of the client’s feelings. Options A and D deny the client’s concerns and provide false reassurance. Option B requests an explanation and does not encourage the expression of feelings.
24. A client with diabetes mellitus has expressed frustration with learning the diabetic regimen and insulin administration. Which of the following is an initial action by the home care nurse?
a)   attempt to identify the cause of the frustration
b)   call the physician to discuss the termination of home-care services
c)   offer to administer the insulin on a daily basis until the client is ready to learn
d)   continue with diabetic teaching, knowing that the client will overcome any frustrations
 A.   The home-care nurse must determine what is causing the client’s frustration. Continuing to teach may only further block the learning process. Administering the insulin provides only a short-term solution. Terminating the client from home-care services achieves nothing and is considered abandonment unless other follow-up care is arranged.
25. A client who has been newly diagnosed with diabetes mellitus has a nursing diagnosis of Ineffective health maintenance related to anxiety regarding the self-administration of insulin. Initially, the nurse should plan to:
a)   teach a family member to give the client the insulin
b)   use an orange for the client to inject into until the client is less anxious
c)   insert the needle, and have the client push in the plunger and remove the needle
d)   give the injection until the client feels confident enough to do so by himself or herself
C.      Some clients find it difficult to insert a needle into their own skin. For these clients, the nurse might assist by selecting the site and inserting the needle. Then, as a first step in self-injection, the client can push in the plunger and remove the needle. Options A and D place the client in a dependent role. Option B is not realistic, considering the subject of the question.

26. The nurse is assessing a client with Addison’s disease for signs if hyperkalemia. The nurse expects to note which of the following if hyperkalemia is present?
a)    polyuria
b)   cardiac dysrhythmias
c)   dry mucous membranes
d)   prolong bleeding time
     B.     The inadequate production of aldosterone in clients with Addison’s disease causes the inadequate excretion of potassium and results in hyperkalemia. The clinical manifestations of hyperkalemia are the result of altered nerve transmission. The most harmful consequence of hyperkalemia is its effect on cardiac function. Options A, C, and D are not manifestations that are associated with Addison’s disease or hyperkalemia.
27. The nurse is admitting a client to the hospital who recently had a bilateral adrenalectomy. Which intervention is essential for the nurse to include in the client’s plan of care?
a) prevent social isolation
b) consider occupational therapy
c) discuss changes in body image
d) avoid stress-producing situations and procedures
 D.   Adrenalectomy can lead to adrenal insufficiency. Adrenal hormones are essential to maintaining homeostasis in response to stressors. Options A, B, and C are not essential interventions specific to this client’s problem.
28. The nurse who is caring for a client with Grave’s disease notes a nursing diagnosis of “imbalanced nutrition: less than body requirements related to the effects of the hypercatabolic state” in the care plan. Which of the following indicates a successful outcome for this diagnosis?
a)         the client verbalizes the need to avoid snacking between meals
b)         the client discusses the relationship between mealtime and the blood glucose level
c)         the client maintains the normal weight or gradually gains weight if it is below normal
d)        the client demonstrates knowledge regarding the need to consume a diet that is high in fat and low in protein
 C.     Graves’ disease causes a state of chronic nutritional and caloric deficiency caused by the metabolic effects of excessive T3 and T4. Clinical manifestations are weight loss and increased appetite. Therefore, it is a nutritional goal that the client will not lose additional weight and that he or she will gradually return to the ideal body weight, if necessary. To accomplish this, the client must be encouraged to eat frequent high-calorie, high-protein, and high-carbohydrate meals and snacks.
29. The nurse assesses the client with a diagnosis of thyroid storm. Which classic signs and symptoms associated with thyroid storm indicate the priority need for immediate nursing intervention?
a)   polyuria, nausea, and severe headache
b)   hypotension, translucent skin, and obesity
c)   fever, tachycardia, and systolic hypertension
d)   profuse diaphoresis, flushing, and constipation
 C.      The excessive amounts of thyroid hormone cause a rapid increase in the metabolic rate, thereby causing the classic signs and symptoms of thyroid storm such as fever, tachycardia, and hypertension. When these signs present themselves, the nurse must take quick action to prevent deterioration of the client’s health because death can ensue. Priority interventions include maintaining a patent airway and stabilizing the hemodynamic status. Options A, B, and D do not indicate the need for immediate nursing intervention.
30. A client is admitted to the hospital for a thyroidectomy. While preparing the client for surgery, the nurse assesses the client for psychosocial problems that may cause preoperative anxiety, knowing that a realistic source of anxiety is fear of:
a)   sexual dysfunction and infertility
b)   imposed dietary restrictions after discharge
c)   developing gynecomastia and hirsutism postoperatively
d)   changes in body image secondary to the location of the incision
D.   Because the incision is in the neck area, the client may be fearful of having a large scar postoperatively. Sexual dysfunction and infertility could possibly occur if the entire thyroid gland was removed and if the client was not placed on thyroid replacement medications. The client will not have specific dietary restrictions after discharge. Having all or part of the thyroid gland removed will not cause the client to experience gynecomastia or hirsutism.


Cardiovascular NCLEX Questions

1. As part of cardiac assessment, the nurse palpates the apical pulse. To perform this assessment, the nurse places the fingertips at which of the following locations?
a) at the left midclavicular line at the fifth intercostal space
b) at the left midclavicular line at the third intercostal space
c) to the right of the midclavicular line at the fifth intercostal space
d) to the right of the midclavicular line at the third intercostal space
 A.  The point of maximal impulse (PMI), where the apical pulse is palpated, is normally located in the  fourth or fifth intercostal space, at the left midclavicular line.
2. A nurse is assessing the client with left-sided heart failure. The client states that he needs to use three pillows under the head and chest at night to be able to breathe comfortably while sleeping. The nurse documents that the client is experiencing:
a) orthopnea
b) dyspnea at rest
c) dyspnea on exertion
d) paroxysmal nocturnal dyspnea
 A.  Dyspnea is a subjective complaint that can range from an awareness of breathing to physical distress and does not necessarily correlate with the degree of heart failure. Dyspnea can be exertional or at rest. Orthopnea is a more severe form of dyspnea, requiring the client to assume a “three-point” position while upright and use pillows to support the head and thorax at night. Paroxysmal nocturnal dyspnea is a severe form of dyspnea occurring suddenly at night because of rapid fluid reentry into the vasculature from the interstitium during sleep.
3. The nurse suspects that a client who had a myocardial infarction is developing cardiogenic shock. The nurse assesses for which peripheral vascular manifestation of this complication?
a) flushed, dry skin with bounding pedal pulses
b) warm, moist skin with irregular pedal pulses
c) cool, clammy skin with weak or thready pedal pulses
d) cool, dry skin with alternating weak and strong peal pulses
 C.    Classic signs of cardiogenic shock include increased pulse (weak and thready), decreased blood pressure, decreasing urinary output, signs of cerebral ischemia (confusion, agitation), and cool, clammy skin.
4. The nurse is caring for a client who returns from cardiac surgery with chest tubes in place. The nurse assesses the drainage on an hourly basis and determines that the client is stable as long as drainage does not exceed how many milliliters over the 24 hours?
a) 100
b) 200
c) 500
d) 1000
 C.    Chest tube drainage should not exceed 100 mL per hour during the first 2 hours postoperatively, and approximately 500 mL of drainage is expected in the first 24 hours after cardiac surgery. The nurse measures and records the drainage on an hourly basis. The drainage is initially dark red and becomes more serous over time.
5. The home health nurse is performing an initial assessment on a client who has arrived home after insertion of a permanent pacemaker. Evaluation of the client’s understanding of self-care is evident if the client reports which of the following?
a) I will never be able to operate a microwave oven again
b) I should expect occasional feelings of dizziness and fatigue
c) I will take my pulse in the wrist or neck daily and record it in a log
d) moving my arms and shoulders vigorously helps to check pacemaker functioning
C.    Clients with permanent pacemakers must be able to take their pulse in the wrist and/or neck accurately in order to note any variation in the pulse rate or rhythm that may need to be reported to the physician. Clients can safely operate most appliances and tools, such as microwave ovens, video recorders, AM-FM radios, electric blankets, lawn mowers, and leaf blowers, as long as the devices are grounded and in good repair. If the client experiences any feelings of dizziness, fatigue, or an irregular heartbeat, the physician is notified. The arms and shoulders should not be moved vigorously for 6 weeks after insertion.
6.  The nurse is applying ECG electrodes to a diaphoretic client. The nurse does which of the following to keep the electrodes from coming loose?
a) secures the electrodes with adhesive tape
b) places clear, transparent dressings over the electrodes
c) applies lanolin to skin before applying the electrodes
d) cleanses the skin with alcohol before applying the electrodes
 D.  Alcohol defats the skin and will help the electrodes adhere to the skin. Placing adhesive tape or a clear dressing over the electrodes will not help the adhesive gel of the actual electrode to make better contact with the diaphoretic skin. Lanolin or any other lotion makes the skin slippery and prevents good initial adherence.
7. The nurse is preparing a client for cardioversion using anterolateral paddle placement. The nurse places the conductive gel pads at which areas on the client’s chest in preparation for this procedure?
a) left fourth intercostal space and left fifth intercostal space at midaxilary line
b) left second intercostal space and left fifth intercostal space at midaxillary line
c) right fourth intercostal space and left fifth intercostal space at anterior axillary line
d) right second intercostal space and left fifth intercostal space at anterior axillary line
 D.   Anterolateral paddle placement for external countershock involves placing one paddle at the right second intercostal space and the other at the fifth intercostal space at the anterior axillary line.
8. The nurse is evaluating the outcomes of care for a client who experienced an acute myocardial infarction. Which of the following findings indicate that an expected outcome for the nursing diagnosis of Decreased Cardiac Output has been met?
a) the client complains of symptoms that require immediate action
b) the client reports absence of dyspnea and anginal pain with activity
c) cardiac output is 3 liters per minute when measured with a pulmonary artery catheter
d) cardiac monitor shows a heart rate of 50 beats per minute after the client has eaten dinner
 B.   Dyspnea and angina are signs of altered cardiac output. The absence of these with activity indicates that cardiac output is adequate. Normal adult cardiac output is 4 to 8 liters per minute. A low heart rate affects cardiac output. The client’s heart rate should be between 60 and 100 beats per minute. Complaints of symptoms that require immediate action are not an expected outcome.
9. The nurse is caring for the client with a dissecting abdominal aortic aneurysm. The nurse avoids which of the following while caring for the client?
a) performs deep palpation of the abdomen
b) auscultates the abdominal mass for bruits
c) tells the client to report abdominal or back pain
d) turns the client to the side to look for ecchymoses on the lower back
 A.   The nurse avoids deep palpation in the client in which a dissecting abdominal aortic aneurysm is known or suspected. Doing so could place the client at risk for rupture. The nurse looks for ecchymoses on the lower back to determine if the aneurysm is leaking and tells the client to report abdominal or back pain, which may radiate to the groin, buttocks, or legs. The nurse should auscultate for bruits.
10. A client has undergone angioplasty of the iliac artery. Which of the following techniques should the nurse perform to best detect bleeding from the angioplasty in the region of the iliac artery?
a) palpate the pedal pulses
b) measure the abdominal girth
c) ask the client about mild pain in the area
d) auscultate over the iliac area with a doppler device
B.     Bleeding after iliac artery angioplasty causes blood to accumulate in the retroperitoneal area. This can most directly be detected by measuring abdominal girth. Palpation and auscultation of pulses determine patency. Assessment of pain is routinely done, and mild regional discomfort is expected.

11. A client is scheduled for a right femoral-politeal bypass graft. The client has a nursing diagnosis of Ineffective Tissue Perfusion. The nurse takes which of the following actions before surgery to address this nursing diagnosis?
a) helps the client void before surgery
b) completes a preoperative checklist
c) marks the location of pedal pulses on the right leg
d) checks the results of any baseline coagulation studies
C.  A nursing diagnosis of Ineffective tissue perfusion in the client scheduled for femoral-popliteal bypass grafting indicates that the client is likely to have diminished peripheral pulses. It is important to mark the location of any pulses that are palpated or auscultated. This provides a baseline for comparison in the postoperative period. The other options are part of routine preoperative care.
12.  A client who underwent peripheral arterial bypass surgery 16 hours ago complains of increasing pain in the leg at rest, which worsens with movement and is accompanied by paresthesias. The nurse should take which of the following actions?
a) call the physician
b) administer an opioid analgesic
c) apply warm moist heat for comfort
d) apply ice to minimize and developing swelling
 A.      The classic signs of compartment syndrome are pain at rest that intensifies with movement and the development of paresthesias. Compartment syndrome is characterized by increased pressure within a muscle compartment caused by bleeding or excessive edema. It compresses the nerves in the area and can cause vascular compromise. The physician is notified immediately because the client could require an emergency fasciotomy. Options B, C, and D are incorrect actions.
13.  The nurse in an ambulatory care clinic takes a client’s blood pressure (BP) in the left arm and notes that it is 200/118 mmHg. Which action should the nurse implement next?
a) notify the physician
b) inquire about the presence of kidney disorders
c) check the client’s blood pressure in the right arm
d) recheck the pressure in the same arm within 30 seconds
 C.    When a high BP reading is noted, the nurse takes the pressure in the opposite arm to see if the blood pressure is elevated in one extremity only. The nurse would also recheck the blood pressure in the same arm, but would wait at least 2 minutes between readings. The nurse would inquire about the presence of kidney disorders that could contribute to the elevated blood pressure. The nurse would notify the physician because immediate treatment may be required, but this would not be done without obtaining verification of the elevation.
14. Acetylsalicylic acid (aspirin) is prescribed for a client with coronary artery disease before a percutaneous transluminal coronary angioplasty (PTCA). The nurse administers the medication, knowing that it is prescribed to:
a) relieve postoperative pain
b) prevent thrombus formation
c) prevent postoprocedure hyperthermia
d) prevent inflammation of the puncture site
 B.   Before PTCA, the client is usually given an anticoagulant, commonly aspirin, to help reduce the risk of occlusion of the artery during the procedure because the aspirin inhibits platelet aggregation. Options A, C, and D are unrelated to the purpose of administering aspirin to this client.
15. The nurse develops a plan of care for a client admitted to the hospital with a diagnosis of an acute myocardial infarction (MI). The priority nursing diagnosis in the acute phase would be:
a) anxiety
b) pain, acute
c) powerlessness
d) interrupted family processes
B.    Pain is the prevailing symptom of acute MI. Relief of pain is a priority. Pain stimulates the autonomic nervous system, increasing myocardial oxygen demand. Although options A, C, and D may also be appropriate nursing diagnoses, the presence of pain impacts on these additional nursing diagnoses.
16. A coronary care unit (CCU) nurse is caring for a client admitted with acute myocardial infarction (MI). The nurse monitors for which most common complication of MI?
a) cardiogenic shock
b) cardiac dysrhythmias
c) congestive heart failure
d) recurrent myocardial infarction
 B.  Dysrhythmias are the most common complication and cause of death after an MI. Cardiogenic shock, congestive heart failure, and recurrent MI are also complications but occur less frequently.
17. A client has a nursing diagnosis of Excess Fluid Volume listed on the care plan. Which of the following assessment findings supports continued use of this nursing diagnosis?
a) weak pulse
b) bibasilar crackles
c) decreased blood pressure
d) flat neck veins with the head of the bed at 45 degrees
 B. Signs of excess fluid volume include bounding pulse, elevated blood pressure, crackles or other adventitious breath sounds, edema of the sacrum or lower extremities, and neck vein distention with the head of the bed positioned at a 45-degree angle. Other signs may include changes in level of consciousness if fluids shifts are occurring.
18. A client is experiencing acute cardiac and cerebral symptoms related to excess fluid volume. The nurse implements which of the following measures to increase the client’s comfort until specific therapy is ordered by the physician?
a) measures urine output on a hourly basis
b) measures intravenous and oral fluid intake
c) elevates the client’s head to at least 45 degrees
d) administers oxygen at 4 liters per minute by nasal cannula
 C.  Elevating the head of the bed to 45 degrees decreases venous return to the heart from the lower body, thus reducing the volume of blood that has to be pumped by the heart. It also promotes venous drainage from the brain, reducing cerebral symptoms. Oxygen is a medication and is not administered without an order. Intake and output should be monitored and recorded to provide current information about the client’s volume status. Options A, B, and D are important measures, although they do not improve the client’s comfort.
19. The nurse is performing an admission assessment for a client admitted to the hospital with a diagnosis of Raynaud’s disease. The nurse assesses for the symptoms associated with Raynaud’s disease by:
a) checking for a rash on the digits
b) observing for softening of the nails or nail beds
c) palpating for a rapid or irregular peripheral pulse
d) palpating for diminished or absent peripheral pulses
D.   Raynaud’s disease produces closure of the small arteries in the distal extremities in response to cold, vibration, or external stimuli. Palpation for diminished or absent peripheral pulses checks for interruption of circulation. The nails grow slowly, become brittle or deformed, and heal poorly around the nail beds when infected. Skin changes include hair loss, thinning or tightening of the skin, and delayed healing of cuts or injuries. Although palpation of peripheral pulses is correct, a rapid or irregular pulse would not be noted. Peripheral pulses may be normal, absent, or diminished.
20. An 18 year old client seeks medical attention for intermittent episodes in which the fingers of both hands become cold, pale, and numb, followed by redness and swelling and throbbing, achy pain. Raynaud’s disease is suspected. The nurse further assesses the client to see if these episodes occur with:
a) exposure to heat
b) being in a relaxed environment
c) ingestion of coffee or chocolate
d) prolonged episodes of inactivity
C.  Raynaud’s disease is a bilateral form of intermittent arteriolar spasm, which can be classified as obstructive or vasospastic. Episodes are characterized by pallor, cold, numbness, and possible cyanosis of the fingers, followed by erythema, tingling, and aching pain. Attacks are triggered by exposure to cold, nicotine, caffeine, trauma to the fingertips, and stress. Prolonged episodes of inactivity are unrelated to these episodes.

21. The nurse is beginning to ambulate a client with a nursing diagnosis of Activity Intolerance who was admitted to the hospital for bacterial endocarditis. The nurse evaluates that the client is best tolerating the exercise if which of the following parameters is noted by the nurse?
a) mild dypnea after walking 10 feet
b) minimal chest pain rated “1″ on the pain scale
c) pulse rate that increases form 68 to 98 beats per minute
d) blood pressure that increases form 114/82 to 118/86 mm?Hg
 D.   General indicators that a client is tolerating exercise include an absence of chest pain or dyspnea, a pulse rate increase of less than 20 beats per minute, and a blood pressure change of less than 10 mm Hg.
22. Cardiac monitoring leads are placed on a client who is at risk for premature ventricular contractions (PVCs). The nurse assesses the client’s rhythm to detect PVCs by looking for:
a) wide and bizarre QRS complexes
b) a P wave preceding every QRS complex
c) QRS complexes that are short and narrow
d) inverted P waves before the QRS complexes
 A.   PVCs are abnormal ectopic beats originating in the ventricles. They are characterized by an absence of P waves, wide and bizarre QRS complexes, and a compensatory pause that follows the ectopy.
23. A client with angina pectoris is receiving isosorbide mononitrate (Imdur) to promote vasodilation. The client states a dislike for the medication because it causes a headache. The nurse makes which of the following interpretations about the client’s statement?
a) this is a common but unhealthy response to the medication
b) this response is caused by cerebral hypoxia induces by the medication
c) this is an extremely adverse reaction and should be reported to the physician
d) this is a common response that will diminish as tolerance to nitroglycerin increases
 D.  Headache is a common side effect of nitroglycerin because of its vasodilator properties. The incidence of headache diminishes over time as the client develops tolerance to the medication. The client should be encouraged to continue its use as needed and to take acetaminophen (Tylenol) or aspirin for headache, according to the preference of the prescribing physician.
24. A client is experiencing pulmonary edema as an exacerbation of chronic left-sided heart failure. The nurse assesses the client for which of the following manifestations?
a) weight loss
b) bilateral crackles
c) distended neck veins
d) peripheral pitting edema
 B.  The client with pulmonary edema presents primarily with symptoms that are respiratory in nature, because the blood flow is stagnant in the lungs, which lie behind the left side of the heart from a circulatory standpoint. The client would experience weight gain from fluid retention, not weight loss. Distended neck veins and peripheral pitting edema are classic signs of right-sided heart failure.
25. A client is admitted to the cardiac intensive care unit following cardiac surgery. The nurse notes that in the first hour after admission, the mediastinal chest tube drainage was 75 ml. During the second hour, the drainage has dropped to 5 ml. The nurse interprets that:
a) this is normal
b) the tube may be occluded
c) the lung has fully reexpand
d) the client needs to cough and deep breath
B.   Chest tube drainage should not exceed 100 mL per hour during the first 2 hours postoperatively, and approximately 500 mL of drainage is expected in the first 24 hours after cardiac surgery. The sudden drop in drainage between the first and second hour indicates that the tube is possibly occluded and requires further assessment by the nurse. Options A, C, and D are incorrect interpretations.

26. The nurse teaches a client with thromboangitis obliterans (Buerger’s disease) about measures to control the disease process. The nurse determines that the client needs further instructions about these measures if the client states which of the following?
a) I need to stop smoking immediately
b) I need to keep my legs and arms cool
c) I will need to take nifedipine (Procardia) as directed
d) I need to watch for signs and symptoms of skin breakdown
 B.   Interventions are directed at preventing the progression of thromboangiitis obliterans and include conveying the need for immediate smoking cessation, providing medications prescribed for vasodilation, such as nifedipine (Procardia), a calcium channel blocker, or prazosin (Minipress), an a-adrenergic blocker. The client should maintain warmth to the extremities, especially by avoiding exposure to cold. The client should inspect the extremities and report signs of infection or ulceration.
27. The nurse notes bilateral +2 edema in the lower extremities of a client with known coronary artery disease who was admitted to the hospital 2 days ago. The nurse plans to do which of the following next after noting this finding?
a) order daily weights starting on the following morning
b) review the intake and output records for the last 2 days
c) request a sodium restriction of 1g per day from the physician
d) change the time of diuretic administration from morning to evening
 B.  Edema is the accumulation of excess fluid in the interstitial spaces, which can be measured by intake greater than output and by a sudden increase in weight (2.2 pounds = 1 kilogram). Obtaining a weight on the following day does not provide the nurse with immediate information. In addition, the nurse would need to evaluate serial weights for comparison. Strict sodium restrictions are reserved for clients with severe symptoms. Diuretics should be administered in the morning whenever possible to avoid nocturia.
28. The nurse in the emergency department is assessing a client with chest pain. Which finding helps determine that the pain is caused by myocardial infarction (MI)?
a) the client experienced no nausea or vomiting
b) the pain was described as burning and gnawing
c) the client reports that the pain began while pushing a lawnmower
d) the pain, unrelieved by nitroglycerin, was relieved with morphine sulfate
B.   The pain of angina may radiate to the left arm, is often precipitated by exertion or stress, has few associated symptoms, and is relieved by rest and nitroglycerin. The pain of MI may radiate to the left arm, shoulder, jaw, and neck. It typically begins spontaneously, lasts longer than 30 minutes, is frequently accompanied by associated symptoms (nausea, vomiting, dyspnea, diaphoresis, anxiety), and requires opioid analgesics for relief. A burning and gnawing pain is more likely noted in an upper gastrointestinal disorder.
29. The nurse is assessing a client who has been hospitalized with acute pericarditis. The nurse monitors the client for cardiac tamponade, knowing that which of the following is a manifestation of this complication of pericarditis?
a) bradycardia
b) paradoxical pulse
c) bounding heart sounds
d) flattened jugular veins
 B.   Assessment findings with cardiac tamponade include tachycardia, distant or muffled heart sounds, jugular vein distention, and a falling blood pressure (BP), accompanied by paradoxical pulse (a drop in inspiratory BP by greater than 10 mm Hg).
30. The nurse is assisting with positioning the client for pericardiocentesis to treat cardiac tamponade. How should the nurse position the client?
a) supine with slight Trendelenberg’s position
b) lying on right side with a pillow under the head
c) lying on the left side with a pillow under the chest wall
d) supine with the head of bed elevated at a 30 to 60 degree angle
D.    The client undergoing pericardiocentesis is positioned supine with the head of the bed raised to a 30- to 60-degree angle. This places the heart in proximity to the chest wall for easier insertion of the needle into the pericardial sac. Options A, B, and C are incorrect positions.

36. A physician writes an order to obtain a 12-lead ECG on a client, and the nurse informs the client of the procedure. Which client statement indicates that the client understands the procedure?
a) I should not breathe while the ECG is running
b) I need to lie still while the ECG is being done
c) I must take a deep breath when the ECG begins
d) If I move when the ECG begins, I will be shocked
 B.     Good contact between the skin and electrodes is necessary to obtain a clear 12-lead ECG tracing. Therefore, the electrodes are placed on the flat surfaces of the skin just above the ankles and wrists. Movement may cause a disruption in that contact. The client does not need to hold his breath or take a deep breath during the procedure. The client needs to be reassured that a shock will not be received. Options A, C, and D are incorrect client statements.
37. A client will repeated episodes of pulmonary emboli from thromboembolism is scheduled for insertion of an inferior vena cava filter (Greenfield filter). The nurse determines that the client has an adequate understanding of the procedure if the client makes which of the following statements?
a) the filter will keep new blood clots from forming in my legs
b) it’s too bad I have to continue anticoagulant therapy after the surgery
c) the filter will be like a catcher’s mitt and keep the clots from going to my lungs
d) I don’t mind having a filter in my artery if it means I won’t have any more trouble
 C.     Insertion of an inferior vena cava filter is indicated for clients with recurrent deep vein thrombosis and/or pulmonary emboli who do not respond to medical therapy, and when anticoagulant therapy is ineffective or contraindicated. The filter device or “umbrella” is inserted percutaneously in the inferior vena cava, where it springs open and attaches to the vena caval wall. The device has holes to allow blood flow, but traps larger clots, thus preventing pulmonary emboli. The filter does not prevent blood clots from forming and is not placed in an artery. Vena cava filters are less effective than anticoagulation and may lead to deep vein thrombosis, so they are generally used only when anticoagulant therapy is ineffective or contraindicated.
38. A client is admitted to the hospital with a diagnosis of infective endocarditis from Streptococcus viridans. The client asks the nurse about the antibiotic therapy that will be given. Knowing that the client has no medication allergies, the nurse prepares the client to receive:
a) amphotericin B (Fungizone) IV for 10 days, followed by oral doses for 3 weeks
b) amphotericin B (Fungizone) IV for 4 to 6 weeks, continuing at home after hospital discharge
c) penicillin G benzathine (Bicillin L-A) IV for 4 to 6 weeks, continuing at home after hospital discharge
d) penicillin G benzathine (Bicillin L-A) IV for 10 days, followed by oral doses for 2 weeks
 C.  Penicillin is frequently the medication of choice for treating endocarditis of bacterial origin. The standard duration of therapy is 4 to 6 weeks, with home care support after hospital discharge, which is usually in 7 to 10 days. Amphotericin B is an antifungal agent and would not be effective with this type of infection.
39. A client has a slow, regular pulse. On the monitor, the nurse notes regular QRS complexes with no P waves and a ventricular rate of 50 beats per minute. The nurse suspects that there is a problem at which part of the cardiac conduction system?
a) the left ventricle
b) the bundle of His
c) the sinoatrial (SA) node
d) the atriovnentricular (AV) node
 C.   A normal P wave indicates that the impulse that depolarized the atrium was initiated in the SA node. A change in the form (inverted P wave) or the absence of a P wave can indicate a problem at this part of the conduction system, with the resulting impulse originating from an alternate site lower in the conduction pathway. Options A, B and D are incorrect.
40. A cleint remains in atrial fibrillation with rapid ventricular response despite pharmacological intervention. Synchronous cardioversion is scheduled to convert the rapid rhythm. The nurse plans to implement which important action to ensure safety and prevent complications of this procedure?
a) cardiovert the client at 360 joules
b) sedate the client before cardioversion
c) ensure that emergency equipment is available
d) ensure that the defibrillator is set on the synchronous mode

41. The nurse is teaching a client with cardiomyopathy about home care safety measures. The nurse provides instructions on which most important measure to ensure client safety?
a) reporting pain
b) taking vasodilators
c) avoiding over-the-counter medications
d) moving slowly from a sitting to a standing position
 D.  Orthostatic changes can occur in the client with cardiomyopathy as a result of venous return obstruction. Sudden changes in blood pressure may lead to falls. Vasodilators are not normally prescribed for the client with cardiomyopathy. Options A and C, although important, are not directly related to the issue of safety.
42. The nurse instructs a client with a diagnosis of atrial fibrillation to use an electric razor for shaving. The nurse tells the client that the importance of its use is that:
a) cuts need to be avoided
b) any cut may cause infection
c) electric razors can be disinfected
d) all straight razors contain bacteria
 A.  Clients with atrial fibrillation are placed on anticoagulants to prevent thrombus formation and possible stroke. The importance of use of an electric razor is to prevent cuts and possible bleeding. Options B, C, and D are all unrelated to the subject of bleeding; rather, they relate to infection.
43. A cardiac catheterization, using the femoral artery approach, is performed to assess the degree of coronary artery thrombosis in a client. Which nursing action following the procedure is unsafe for the client?
a) encouraging the client to increase fluid intake
b) placing the client’s bed in the fowler’s position
c) resuming prescribed precatheterization medications
d) instructing the client to move the toes when checking circulation, motion, and sensation
 B.   Immediately following a cardiac catheterization with the femoral artery approach, the client should not flex or hyperextend the affected leg to avoid blood vessel occlusion or hemorrhage. Placing the client in the Fowler’s position (flexion) increases the risk of occlusion or hemorrhage. Fluids are encouraged to assist in removing the contrast medium from the body. The precatheterization medications are needed to treat acute and chronic conditions. Asking the client to move the toes is done to assess motion, which could be impaired if a hematoma or thrombus was developing.
44. The nurse is receiving a client being transferred from the postanesthesia care unit following an above-the-knee amputation. The nurse should take which action to safely position the client at this time?
a) elevate the foot of the bed
b) position the residual limb flat on the bed
c) put the bed in reverse trendelenburg’s position
d) keep the residual limb flat with the client lying on the operative side
 A.   Edema of the residual limb is controlled by elevating it on pillows or the foot of the bed for the first 24 hours only after surgery. Following the first 24 hours, the residual limb is usually placed flat on the bed to reduce hip contracture. Edema is also controlled by residual limb wrapping techniques. Reverse Trendelenburg’s position does not provide direct limb elevation.
45. The postmyocardial infarction client is scheduled for a technetium-99m ventriculography (multigated acquisition [MUGA] scan). The nurse ensures that which item is in place before the procedure?
a) a foley catheter
b) signed informed consent
c) a central venous pressure (CVP) line
d) notation of allergies to iodine or shellfish
 B.  MUGA is a radionuclide study used to detect myocardial infarction, decreased myocardial blood flow, and left ventricular function. A radioisotope is injected intravenously. Therefore, a signed informed consent is necessary. A Foley catheter and CVP line are not required. The procedure does not use radiopaque dye; therefore, allergy to iodine and shellfish is not a concern.

46. The nurse has applied the patch electrodes of an automatic external defibrillator (AED) to the chest of a client who is pulseless. The defibrillator has interpreted the rhythm to the ventricular fibrillation. The nurse then:
a) administers rescue breathing during the defibrillation
b) performs cardiopulmonary resuscitation (CPR) for 1 minute before defibrillating
c) charges the machine and immediately pushes the “discharge” buttons on the console
d) orders any personnel away from the client, charges the machine, and defibrillates through the console
 D.   If the AED advises to defibrillate, the nurse or rescuer orders all persons away from the client, charges the machine, and pushes both of the “discharge” buttons on the console at the same time. The charge is delivered through the patch electrodes, and this method is known as “hands-off” defibrillation, which is safest for the rescuer. The sequence of charges (up to three consecutive attempts at 200, 300, and 360 joules) is similar to that of conventional defibrillation. Option A is contraindicated for the safety of any rescuer. Performing CPR delays the defibrillation attempt.
47. The nurse is planning care for a client diagnosed with deep vein thrombosis (DVT) of the left leg who is experiencing severe edema and pain in the affected extremity. Which intervention should the nurse avoid in the care of this client?
a) elevate the left leg
b) apply moist heat to the left leg
c) administer acetaminophen (Tylenol)
d) ambulate in the hall three times per shift
 D.   Management of the client with DVT who is experiencing severe edema and pain includes bed rest; limb elevation; relief of discomfort with warm moist heat and analgesics as needed; anticoagulant therapy; and monitoring for signs of pulmonary embolism. In current practice, activity restriction may not be ordered if the client is receiving low-molecular-weight heparin; however, some physicians may still prefer bed rest for the client.
48. The nurse is planning preoperative care for a client scheduled for insertion of an inferior vena cava filter. The nurse questions the physician about withholding which regularly scheduled medication on the day before surgery?
a) docusate sodium (Colace)
b) furisemide (Lasix)
c) potassium chloride (K-Dur)
d) warfarin sodium (Coumadin)
 D.   In the preoperative period, the nurse consults with the physician about withholding warfarin sodium to avoid the occurrence of hemorrhage. Docusate sodium is a stool softener, furosemide is a diuretic, and potassium chloride is a supplement.
49. A hospitalized client with hypertension has been started on catopril (Capoten). The nurse ensures that the client does which of the following specific to this medication?
a) drinks plenty of water
b) eats foods that are high in potassium
c) takes in sufficient amounts of high-fiber foods
d) moves from a sitting to a standing position slowly
 D.    Orthostatic hypotension is a concern for clients taking antihypertensive medications. Clients are advised to avoid standing in one position for lengthy amounts of time, to change positions slowly, and to avoid extreme warmth (showers, bath, and weather). Clients are also taught to recognize the symptoms of orthostatic hypotension, including dizziness, light-headedness, weakness, and syncope. Options A, B, and C are not specific to this medication.
50. A 24-year old female with a family history of heart disease presents to the physician’s office asking to begin oral contraceptive therapy for birth control. The nurse would next inquire whether the client:
a) exercises regularly
b) is currently a smoker
c) eats a low-cholesterol diet
d) has taken oral contraceptives before
B.      Oral contraceptive use is a risk factor for heart disease, particularly when it is combined with cigarette smoking. Regular exercise and keeping total cholesterol levels less than 200 mg/dL are general measures to decrease cardiovascular risk.




NCLEX Study Course about Renal Disorders

1. A client with a posterior wall bladder injury has had surgical repair and placement of a suprapubic catheter. The nurse plans to do which of the following to prevent complications with the use of this catheter?
a) monitor urine output every shift
b) encourage a high intake of oral fluids
c) prevent kinking of the catheter tubing
d) measure specific gravity once a shift
 C.  A complication after surgical repair of the bladder is disruption of sutures caused by tension on them from urine buildup. The nurse prevents this from happening by ensuring that the catheter is able to drain freely. This involves basic catheter care, including keeping the tubing free from kinks, keeping the tubing below the level of the bladder, and monitoring the flow of urine frequently. Measurement of urine specific gravity and a high oral fluid intake do not prevent complications of bladder surgery. Monitoring urine output every shift is insufficient to detect decreased flow from catheter kinking.


2. A client with benign prostatic hyperplasia undergoes transurethral resection of the prostate (TURP). The nurse orders which of the following solutions from the pharmacy so it is available postoperatively for continuous irrigation (CBI)?
a) sterile water
b) sterile normal saline
c) sterile Dakin’s solution
d) sterile water with 5% dextrose
 B.   Continuous bladder irrigation is done following TURP using sterile normal saline, which is isotonic. Sterile water is not used because the solution could be absorbed systemically, precipitating hemolysis and possibly renal failure. Dakin’s solution contains hypochlorite and is used only for wound irrigation in selected circumstances. Solutions containing dextrose are not introduced into the bladder.
3.   A client with a central venous catheter who is receiving parenteral nutrition (PN) suddenly becomes short of breath, complains of chest pain, and is tachycardic, pale, and anxious. The nurse suspecting an air embolism, places the client n lateral trendelenburg position on the left side and:
a) monitor vital signs every 30 minutes
b) clamps the catheter and notifies the physician
c) slows the rate of the PN after checking the lines for air
d) boluses the client with 500 ml normal saline to break up the air embolism
 B.    If the client experiences air embolus, the nurse should clamp the catheter and notify the physician. The client is also placed in the lateral Trendelenburg position on the left side to trap the air in the right atrium. A fluid bolus would cause the air embolus to travel. Vital signs are monitored continuously.
4.   A client with acute renal failure is ordered to be on a fluid restriction of 1500 ml per day. The nurse best plans to assist the client with maintaining the restriction by:
a) removing the water pitcher from the bedside
b) using mouthwash with alcohol for mouth care
c) prohibiting beverages with sugar to minimize thirst
d) asking the client to calculate the IV fluids into the total daily allotment
 A.  The nurse can help the client maintain fluid restriction through a variety of means. One way is to provide frequent mouth care; however, alcohol-based products should be avoided because they are drying to mucous membranes. The use of ice chips and lip ointments is another intervention that may be helpful to the client on fluid restriction. Beverages that the client enjoys are provided and are not restricted based on sugar content. The client is not asked to keep track of IV fluid intake; this is the nurse’s responsibility. The water pitcher should be removed from the bedside to aid in compliance.
5.   The nurse is caring for a client scheduled to undergo a renal biopsy. To minimize the risk of postprocedure complications, the nurse reports which of the following laboratory results to the physician before the procedure?
a) potassium: 3.8 mEq/L
b) serum creatinine: 1.2 mg/dL
c) prothrombin time: 15 seconds
d) blood urea nitrogen (BUN): 18 mg/dL
C.           Postprocedure hemorrhage is a complication after renal biopsy. Because of this, prothrombin time is assessed before the procedure. The normal prothrombin time range is 11 to 12.5 seconds. The nurse ensures that these results are available and reports abnormalities promptly. Options 1, 2, and 4 identify normal values. The normal BUN is 5 to 20 mg/dL, the normal serum creatinine is 0.6 to 1.3 mg/dL, and the normal potassium is 3.5 to 5.1 mEq/L.
6.   A client with acute pyelonephritis has nausea and vomiting and is scheduled for an intravenous pyelogram. The nurse places highest priority on which action?
a) ask the client to sign the informed consent
b) explain the procedure thoroughly to the client
c) place the client on hourly intake and output measurements
d) request an order for an intravenous infusion from the physician
 D.     The highest priority of the nurse would be to request an order for an intravenous infusion. This is needed to replace fluid lost with vomiting, will be necessary for dye injection for the procedure, and will assist with the elimination of the dye following the procedure. The intake and output should be measured, but this will not assist in preventing dehydration. Explanation of the procedure and obtaining the signed informed consent are done once the client’s physiological needs are met.
7.  A client with urolithiasis is being evaluated to determine the type of stone that is being formed. The nurse plans to keep which of the following items available in the client’s room to assist in this process?
a) a strainer
b) a calorie count sheet
c) a vital signs graphic sheet
d) an intake and output record
 A.    The urine is strained until the stone is passed and obtained and analyzed. Straining the urine will catch small stones that may be sent to the laboratory for analysis. Once the type of stone is determined, an individualized plan of care for prevention and treatment is developed. Options B, C, And D are unrelated to the question.
8.  The nurse is preparing to care for a client following ureterolithotomy who has a ureteral catheter in place. The nurse plans to implement which action in the management of this catheter when the client arrives from the recovery room?
a) clamps the catheter
b) places tension on the catheter
c) checks the drainage from the catheter
d) irrigates the catheter using 10ml sterile normal saline
 C.   Drainage from the ureteral catheter should be checked when the client returns from the recovery room and at least every 1 to 2 hours thereafter. The catheter drains urine from the renal pelvis, which has a capacity of 3 to 5 mL. If the volume of urine or fluid in the renal pelvis increases, tissue damage to the pelvis will result from pressure. Therefore, the ureteral tube is never clamped. Additionally, irrigation is not performed unless there is a specific physician’s order to do so.
9. A client is undergoing hemodialysis and becomes hypotensive. The nurse avoids taking which of the following contraindicated actions?
a) raising the client’s legs and feet
b) preparing to administer a 250 ml normal saline bolus
c) increasing the blood flow from the client into the dialyzer
d) checking the client’s weight and reassessing blood pressure
 C.  To treat hypotension during hemodialysis, the nurse raises the client’s feet and legs to enhance cardiac return. A normal saline bolus of up to 500 mL may be given to increase circulating volume. The nurse would check the client’s weight and reassess the blood pressure. Finally, the transmembrane hydrostatic pressure or the blood flow rate into the dialyzer may be decreased. All of these measures should improve the circulating volume and blood pressure.
10. A nurse is caring for a client with a diagnosis of urolithiasis. The nurse instructs the client that it is most important to perform which of the following activities?
a) record weight every day
b) restrict physical activities
c) strain all urine from each voiding
d) turn, cough, and deep breathe every 2 hours
C.     Obstruction of the urinary tract is the primary problem associated with urolithiasis. Stones recovered from straining urine can be analyzed and can provide direction for prevention of further stone formation. Activities should not be restricted. Options A and D are not specifically related to the client’s diagnosis.
11. A nurse notes that the client’s urinalysis report contains a notation of positive red blood cells (RBC’s). The nurse interprets that this finding is unrelated to which of the following items that is part of the client’s clinical picture?
a) diabetes mellitus
b) history of kidney stones
c) concurrent anticoagulant therapy
d) history of recent blow to the right flank
 A.   Hematuria can be caused by trauma to the kidney, such as with blunt trauma to the lower posterior trunk or flank. Kidney stones can cause hematuria because they scrape the endothelial lining of the urinary system. Anticoagulant therapy can cause hematuria as an adverse effect. Diabetes mellitusdoes not cause hematuria, although it can lead to renal failure from prerenal causes.
12. The nurse has an order to ambulate a client with a nephrostomy tube in the hall four times a day. The nurse determines that the safest way to accomplish this while maintaining the integrity of the nephrostomy tube is to:
a) change the drainage bag to a leg collection bag
b) tie the drainage bag to the client’s waist while ambulating
c) use a walker to hang the drainage bag from while ambulating
d) tell the client to hold the drainage bag higher than the level of the bladder
 A.   The safest approach to protect the integrity and safety of the nephrostomy tube with a mobile client is to attach the tube to a leg collection bag. This allows for greater freedom of movement, while preventing accidental disconnection or dislodgment. The drainage bag is kept below the level of the bladder. Option 3 presents the risk of tension or pulling on the nephrostomy tube by the client during ambulation.
13. A client newly diagnosed with polycystic kidney disease has just finished speaking with the physician about the disorder. The client asks the nurse to explain again what the most serious complication of the disorder might be. In formulating a response, the nurse incorporates the understanding that the most serious complication is:
a) diabetes insipidus
b) end-stage renal disease (ESRD)
c) chronic urinary tract infection (UTI)
d) syndrome of inappropriate antidiuretic hormone (SIADH) secretion
 B.  The most serious complication of polycystic kidney disease is ESRD, which would be managed with dialysis or transplant. There is no reliable way to predict who will ultimately progress to ESRD. Chronic UTIs are the most common complication because of the altered anatomy of the kidney and from development of resistant strains of bacteria. Diabetes insipidus and SIADH secretion are unrelated disorders.
14. A nurse is assigned to care for a client who has returned to the nursing unit following left nephrectomy. The nurse places highest priority on obtaining which of the following assessments?
a) temperature
b) hourly urine output
c) ability to turn side to side
d) tolerance for sips of clear fluids
 B.   Following nephrectomy, it is imperative to measure the urine output on an hourly basis. This is done to  monitor the effectiveness of the remaining kidney and to detect renal failure early, if it should occur. The client may also experience significant pain after this surgery, which could impact on the client’s ability to reposition, cough, and deep breathe. Therefore, the next most important measurements are vital signs (including temperature), pain level, and bed mobility. Clear liquids are not given until the client has bowel sounds.
15. The home care nurse is assessing a client who has begun using peritoneal dialysis. The nurse determines that which manifestation noted in the client would most likely indicate the onset of peritonitis?
a) cloudy dialysate output
b) temperature of 99.0 F oral
c) presence of crystals in dialysate output
d) history of gastrointestinal (GI) upset 1 week ago
A.    Typical symptoms of peritonitis include fever, nausea, malaise, rebound abdominal tenderness, and cloudy dialysate output. The very slight temperature elevation in option 2 is not the clearest indicator of infection. Peritonitis would cause cloudy dialysate but would not cause crystals to appear in the dialysate. The complaint of GI upset is too vague to indicate peritonitis. 

16. The nurse working on a medical-surgical nursing unit is caring for several clients with renal failure. The nurse interprets that which of the following clients is best suited for peritoneal dialysis as a treatment option?
a) a client with severe congestive heart failure
b) a client with a history of ruptured diverticula
c) a client with a history of herniated lumbar disk
d) a client with a history of three previous abdominal surgeries
 A   Peritoneal dialysis may be the treatment option of choice for clients with severe cardiac disease because there is less cardiac stress with this treatment. Severe cardiac disease can be worsened by the rapid shifts in fluid, electrolytes, urea, and glucose that occur with hemodialysis. For the same reason, peritoneal dialysis may be indicated for the client with diabetes mellitus. Contraindications to peritoneal dialysis include diseases of the abdomen such as ruptured diverticula or malignancies; extensive abdominal surgeries; history of peritonitis; obesity; and those with a history of back problems, which could be aggravated by the fluid weight of the dialysate. Severe disease of the vascular system may also be a contraindication.


17. A client undergoing long-term peritoneal dialysis at home is currently experiencing a problem with reduced outflow from the dialysis catheter. The home care nurse inquires whether the client has had a recent problem with which of the following?
a) diarrhea
b) vomiting
c) flatulence
d) constipation
 D.   Reduced outflow may be caused by catheter position and adherence to the omentum, infection, or constipation. Constipation may contribute to reduced outflow in part because peristalsis seems to aid in drainage. For this reason, bisacodyl (Dulcolax) suppositories are sometimes used prophylactically, even without a history of constipation. The other options are unrelated to impaired catheter drainage.
18. A client with a history of heart failure who is undergoing peritoneal dialysis has developed crackles in the lower lung fields. The nurse interprets that this finding is most likely related to:
a) natural progression of the renal failure
b) compliance with dietary sodium restriction
c) intake greater than output on the dialysis record
d) adherence to digoxin (Lanoxin) therapy schedule
 C.  Crackles in the lung fields of the peritoneal dialysis client result from overhydration or from insufficient fluid removal during dialysis. An intake that is greater than the output of peritoneal dialysis fluid would overhydrate the client, resulting in lung crackles. Adherence to medication and diet therapy should control this sign, not exacerbate it. If dialysis is effective, there is no connection between the progression of renal failure and the development of signs of overhydration.
19. The nurse is caring for a client who has returned from the postanesthesia care unit following prostatectomy. The client has a three-way foley catheter with infusion of continuous bladder irrigation solution. The nurse assesses that the flow rate is adequate if the color of the urinary drainage is:
a) clear as water
b) dark cherry colored
c) pale yellow or slightly pink
d) concentrated yellow with small clots
 C.  The infusion of bladder irrigant is not at a preset rate, but rather it is increased or decreased to maintain urine that is a clear, pale yellow color or that has just a slight pink tinge. The infusion rate should be increased if the drainage is cherry colored or if clots are seen. Correspondingly, the rate can be slowed down slightly if the returns are as clear as water.
20. The nurse is assessing a client who is at risk of developing acute renal failure (ARF). The nurse would become most concerned if which of the following assessments was made?
a) urine output 60 ml/hr for last 3 hours, BUN 40 mg/dL, creatinine 1.1 mg/dL
b) urine output 40 ml/hr for last 3 hours, BUN 15 mg/dL, creatinine 0.8 mg/dL
c) urine output 20 ml/hr for last 3 hours, BUN 35 mg/dL, creatinine 2.1 mg/dL
d) urine output 30 ml/hr for last 3 hours, BUN 10 mg/dL, creatinine 1.2 mg/dL
C.      With acute renal failure, the client is often oliguric or anuric, although the client may have nonoliguric renal failure. The BUN and serum creatinine levels also rise, indicating impaired kidney function. Normal serum BUN levels are usually 5 to 20 mg/dL. Normal creatinine levels range from 0.6 to 1.3 mg/dL. The client who has the greatest abnormality in urine output and laboratory values is the client in option 3. This is the client who is most at risk for developing renal failure.
21. A client with acute renal failure has been treated with sodium polystyrene sulfonate (Kayexalate) by mouth. The nurse would evaluate this therapy as effective if which of the following values was noted on follow-up laboratory testing?
a) calcium: 9.8 mg/dL
b) sodium: 142 mEq/L
c) potassium: 4.9 mEq/L
d) phosphorus: 3.9 mg/dL
 C.    Of all the electrolyte imbalances that accompany renal failure, hyperkalemia is the most dangerous because it can lead to cardiac dysrhythmias and death. If the potassium level rises too high, sodium polystyrene sulfonate (Kayexalate) may be administered to cause excretion of potassium through the gastrointestinal tract. Each of the electrolyte levels noted in the options falls within the normal reference range for that electrolyte. The potassium level, however, is measured following administration of this medication to note the extent of its effectiveness.
22. The nurse is admitting a client with chronic renal failure to the nursing unit. The nurse assesses for which most frequent cardiovascular sign that occurs in the client with chronic renal failure?
a) bradycardia
b) tachycardia
c) hypotension
d) hypertension
 D.   Hypertension is the most common cardiovascular finding in the client with chronic renal failure. It is caused by several mechanisms, including volume overload, renin-angiotensin system stimulation, vasoconstriction from sympathetic stimulation, and the absence of prostaglandins. Hypertension may also be the cause of the renal failure. It is an important item to assess because hypertension can lead to heart failure in the chronic renal failure client, because of increased cardiac workload in conjunction with fluid overload. The client may experience tachycardia or bradycardia or may have a normal pulse rate; these cardiovascular manifestations will depend on a variety of physiological events such as fluid overload, fluid deficit, or normal fluid volume.
23. The nurse is assessing the renal function of a client. After directly noting urine volume and characteristics, the nurse assesses which item as the best indirect indicator of renal function?
a) pulse rate
b) blood pressure
c) bladder distention
d) level of consciousness
 B.    The kidneys normally receive 20% to 25% of the cardiac output, even under conditions of rest. In order for kidney function to be optimal, adequate renal perfusion is necessary. Perfusion can best be estimated by the blood pressure, which is an indirect reflection of the adequacy of cardiac output. The pulse rate affects the cardiac output, but it can be altered by factors unrelated to kidney function. Bladder distention reflects a problem or obstruction that is most often distal to the kidneys. Level of consciousness is an unrelated item.
24. A female client with a history of chronic infection of the urinary system complains of burning and urinary frequency. To determine whether the current problem is of renal origin, the nurse would assess whether the client has pain or discomfort in the:
a) urinary meatus
b) suprapubic area
c) pain in the labium
d) right or left costovertebral angle
 D.    Pain or discomfort from a problem that originates in the kidney is felt at the costovertebral angle on the affected side. Bladder infection is often accompanied by suprapubic pain and pain or burning at the urinary meatus when voiding. Ureteral pain is felt in the ipsilateral labium in the female client or the ipsilateral scrotum in the male client.
25. A client tells the nurse about a pattern of getting a strong urge to void, which is followed by incontinence before the client can get to the bathroom. The nurse formulates which of the following nursing diagnoses for this client?
a) urge urinary incontinence
b) stress urinary incontinence
c) total urinary incontinence
d) reflex urinary incontinence
A.       Urge incontinence occurs when the client has urinary incontinence soon after experiencing urgency. Stress incontinence occurs when the client voids in increments that are less than 50 mL and has increased abdominal pressure. Total incontinence occurs when there is an unpredictable and continuous loss of urine. Reflex incontinence occurs when incontinence occurs at rather predictable times that correspond to when a certain bladder volume is attained.


26. The nurse is caring for a client who returned to the nursing unit following suprapubic prostatectomy. The nurse monitors the continuous bladder irrigation to detect which of the following signs of catheter blockage?
a) drainage that is pale pink
b) drainage that is bright red
c) true urine output of 50 ml/hr
d) urine leakage around the three-way catheter at the meatus
 D.   Catheter blockage or occlusion by clots following prostatectomy can result in urine backup and leakage around the urethral meatus. This would be accompanied by a stoppage of outflow through the catheter into the drainage bag. Drainage that is bright red indicates that the irrigant is running too slowly; drainage that is pale pink indicates sufficient flow. A true urine output of 50 mL per hour indicates catheter patency.
27. The nurse is assigned to a client returning from the postanesthesia care unit following transurethral prostatectomy. The nurse avoids doing which of the following after this procedure?
a) reporting signs of confusion
b) monitoring hourly urine output
c) removing the traction tape on the three-way catheter
d) administering a B&O (belladonna and opium) suppository at room temperature
 C.  The nurse avoids removing the traction tape applied by the surgeon in the operating room. The purpose of this tape is to place pressure on the prostate and reduce hemorrhage. B&O suppositories, ordered on a prn basis for bladder spasm, should be warmed to room temperature before administration. The nurse routinely monitors hourly urine output because the client has a three-way bladder irrigation running. The nurse also assesses for confusion, which could result from hyponatremia secondary to the hypotonic irrigant used during the surgical procedure.
28. The nurse has prepared a client for an intravenous pyelogram. The nurse determines that the client understands the procedure if the client states to report which sensation immediately, if it occurs during the procedure?
a) nausea
b) difficulty breathing
c) salty taste in the mouth
d) momentary episode of shivering
 B.    Intravenous pyelography is a contrast study of the kidneys to determine a variety of disorders of the kidneys, ureters, and bladder. Normal sensations during injection of the iodine-based radiopaque dye include a warm, flushed feeling, salty taste in the mouth, and transient nausea. Difficulty breathing, wheezing, hives, or itching indicate an allergic response and should be reported to the physician immediately. This complication is prevented by inquiring about allergies to iodine or shellfish before the procedure.
29. The nurse is admitting to the nursing unit a client who has an arteriovenous (AV) fistula in the right arm for hemodialysis. The nurse plans to best prevent injury to the site by implementing which of the following?
a) applying an allergy bracelet to the right arm
b) putting a large note about the access site on the front of the medical record
c) telling the client to inform all caregivers who enter the room about the presence of the access site
d) placing a sign at the bedside that says “No blood pressure (BP) measurements or venipuncture in the right arm”
 D.  There should be no venipunctures or blood pressure measurements in the extremity with a hemodialysis access device. This is commonly communicated to all caregivers by placing a sign at the client’s bedside. Placing a note on the front of the medical record does not ensure that everyone caring for the client is aware of the access device. An allergy bracelet is placed on the client with an allergy. Some health care agencies, however, do have policies that require application of a wrist bracelet of some type to be placed on the client with a hemodialysis access device. The client should not be responsible for informing the caregivers.
30. The nurse manager of a hemodialysis unit is observing a new nurse preparing to begin hemodialysis on a client with renal failure. The nurse manager intervenes if the new nurse carries out which of the following actions?
a) use sterile technique for needle insertion
b) wears full protective clothing such as goggles, mask, gown, and gloves
c) covers the connection site with a bath blanket to enhance extremity warmth
d) puts on a mask and gives one to the client to wear during connection to the machine
C.     The connection site should not be covered, and it should be visible so that the nurse can assess for bleeding, ischemia, and infection at the site during the hemodialysis procedure. Infection is a major concern with hemodialysis. For that reason, the use of sterile technique and the application of a face mask for both the nurse and client are extremely important. It is also imperative that standard precautions be followed, which includes the use of goggles, mask, gloves, and a gown.
31. A client with urolithiasis is scheduled for extracorporeal shock wave lithotripsy. The nurse ensures that the client understands the procedure and tells the client that:
a) there is no pain at all involved with this procedure
b) he will be anesthetized and placed on a water cushion
c) there are no side effects or complications associated with this procedure
d) the procedure involves breaking up the stone by a vibrating needle that is inserted into the urinary tract
 B.   In extracorporeal shock wave lithotripsy, a noninvasive procedure, the client is anesthetized (spinal or general) and placed on a water cushion. Anesthesia is necessary to keep the client very still during the procedure. Shock waves are administered that shatter the stone without damaging the surrounding tissues. The stone is broken into fine sand, which is secreted into the client’s urine within a few days after the procedure. Hematuria is common after the procedure. The presence of clots in the urine needs to be reported to the physician. Clots could indicate a complication such as a hematoma.
32. A client with chronic renal failure has an indwelling peritoneal catheter in the abdomen for peritoneal dialysis. While bathing, the client spills water on the abdominal dressing covering the abdomen. The nurse plans immediately:
a) change the dressing
b) reinforce the dressing
c) flush the peritoneal dialysis catheter
d) scrub the catheter with providine-iodine
A.    Clients with peritoneal dialysis catheters are at high risk for infection. A dressing that is wet is a conduit for bacteria to reach the catheter insertion site. The nurse ensures that the dressing is kept dry at all times. Reinforcing the dressing is not a safe practice to prevent infection in this circumstance. Flushing the catheter is not indicated. Scrubbing the catheter with povidone-iodine is done at the time of connection or disconnection of peritoneal dialysis 
33. The nurse has given the client with a nephrostomy tube instructions to follow after hospital discharge. The nurse determines that the client understands the instructions if the client verbalizes the need to drink at least how many glasses of water per day?
a) 2 to 4
b) 6 to 8
c) 10 to 12
d) 14 to 16
 B.   The client with a nephrostomy tube needs to have adequate fluid intake to dilute urinary particles that could cause calculus and to provide good mechanical flushing of the kidney and the tube. The nurse encourages the client to take in at least 2000 mL of fluid per day, which is roughly equivalent to 6 to 8 glasses of water. Option 1 is an inadequate amount. Options 3 and 4 are amounts that could distend the renal pelvis.
34. A client with nephrolithiasis arrives at the clinic for a follow-up visit. The laboratory analysis of the stone that client passed 1 week ago indicates that the stone is composed of calcium oxalate. On the basis of this analysis, the nurse tells the client to avoid which of the following foods?
a) pasta
b) lentils
c) lettuce
d) spinach
 D.  Many kidney stones are composed of calcium oxalate. Foods that raise urinary oxalate excretion include spinach, rhubarb, strawberries, chocolate, wheat bran, nuts, beets, and tea.
35. A female client is being discharged from the hospital to home with an indwelling urinary catheter after the surgical repair of the bladder after trauma. The nurse determines that the client understands the principles of catheter management if the client states to:
a) cleanse the perineal area with soap and water once a day
b) keep the drainage bag lower that the level of the bladder
c) limit fluid intake so that the bag will not become full so quickly
d) coil the tubing and place it under the thigh when sitting to avoid tugging on the bladde
B.        The perineal area should be cleansed twice daily and after each bowel movement with soap and water. The drainage bag should be lower than the level of the bladder, and the tubing should be free of kinks and compression. Coiling the tubing and placing it under the thigh can compress the tube. Adequate fluid intake is necessary to prevent infection and to provide natural irrigation of the catheter from increased urine flow.

36. The client with prostatitis asks the nurse, “Why do I need to take a stool softener? The problem is with my urine, not my bowels?” The nurse makes which response to the client?
a) this is a standard medication order for anyone with an abdominal problem
b) this will keep the bowel free of feces, which helps decrease the swelling inside
c) being constipated puts you at more risk for developing complications of prostatitis
d) this will help you prevent constipation, because straining is painful with prostatitis
 D.  Stool softeners are ordered for the client with prostatitis to prevent constipation, which can be painful. It has no direct effect on decreasing swelling. Constipation does not cause complications of prostatitis. Stool softeners are not a standard prescription for “anyone with an abdominal problem.”
37. The nurse is conducting a prostate screening clinic and discussing prevention and risk factors for prostate cancer. The nurse determines that a client understands the educational information that was shared if the nurse overhears the client tell another participants that:
a) a low-fiber diet helps to prevent prostate cancer
b) eating foods high in fat is not a risk factor for prostate cancer
c) green and yellow vegetables should be limited in the diet to prevent prostate cancer
d) an annual prostate exam and a prostate-specific antigen (PSA) test should be performed beginning at the age of 50 years
 D.    An annual prostate exam and a PSA test should be performed beginning at the age of 50 years or at the age of 45 years if the client is at high risk for this type of cancer. A low-fat, high-fiber diet diminishes prostate cancer risk. The increased intake of green or yellow vegetables or of the lycopenes contained in tomatoes may be helpful to reduce risk.
38. A client is being discharged to home without an indwelling urinary catheter after prostatectomy. The nurse plans to teach the client which of the following points as part of the discharge teaching?
a) mowing the lawn is allowed after 1 week
b) avoid lifting more than 50 pounds for 4 to 6 weeks after surgery
c) drink at least 15 glasses of water a day to minimize clot formation
d) notify the physician if fever, increased pain, or an inability to void occurs
 D.   The client should notify the physician if there are any signs of infection, bleeding, increased pain, or urinary obstruction. Strenuous activities that could increase intra-abdominal tension are restricted, such as mowing the lawn. Lifting more than 20 pounds is prohibited for 4 to 6 weeks after surgery. The client should take in 6 to 8 glasses of water or nonalcoholic beverages per day to minimize the risk of clot formation.
39. The nurse is teaching a client with acute renal failure to include proteins in the diet that are considered high quality. Which food item should the nurse discourage because it is a low-quality protein source?
a) eggs
b) fish
c) chicken
d) broccoli
 D.   High-quality proteins come from animal sources and include such foods as eggs, chicken, meat, and fish. Low-quality proteins are derived from plant sources and include vegetables and foods made from grains. Because the renal diet is limited in protein, it is important that the proteins ingested are of high quality.
40. The nurse has completed instructions regarding diet and fluid restriction for the client with chronic renal failure. The nurse determines that the client understands the information presented if the client selected which dessert from the dietary menu?
a) jell-O
b) sherbet
c) ice cream
d) angel food cake
D.    Dietary fluid includes anything that is liquid at room temperature. This includes items such as Jell-O, sherbet, and ice cream. For clients on a fluid-restricted diet, it is helpful to avoid “hidden” fluids to whatever extent possible. This allows the client to take in more fluid by drinking, which can help alleviate thirst.






























1 comment: